SJT At A Glance Practice Paper Flashcards

1
Q
  1. An 8-year-old boy comes into A&E with a badly broken
    leg. He needs surgery. You speak to his parents about the
    surgery. They explain that they are Jehovah’s Witnesses and
    they don’t want their son to have a blood transfusion. What
    should you do?
    Rank the following responses from most appropriate (=1) to
    least appropriate (=5):
    A. Inform your consultant about the situation
    B. Consult local hospital guidelines on bloodless procedures
    C. Ignore the parents’ wishes and treat the child in his best
    interests
    D. Tell the parents that it is unlikely that their son will need a
    transfusion and they should agree to surgery
    E. Tell the parents you will see what you can do
A

ANSWER: AEBCD
This question is about respecting and communicating a
patient’s religious views in such a way as to best accommodate
them into clinical care. For more information on
Jehovah’s Witnesses see Chapter 5. In this scenario you are
expected to seek help in a tricky situation.
A. Informing your consultant is the most appropriate option as
they have to make the final decision in this situation. If you
could only choose one of the options in this question, this
would be it. Everything else is subsidiary to recognising that
you should seek help. The consultant will also be able to advise
you on what to tell the parents (E).
E. Telling the parents you will see what you can do is the next
most appropriate option. It is best practice to value guardians’
wishes and respect that they are acting in the best interests of
the child – but you should speak to a senior before you say this.
B. Consulting the local hospital guidelines is the next most
appropriate option as this will further your own learning and
development.
C. Ignoring the parents’ wishes is not appropriate, as they
should be respected. Your consultant could override their decision
acting in the child’s best interests. However, this is an area
that would cause considerable conflict and is best avoided.
D. Telling the parents it is unlikely that their son will need a
transfusion is the least appropriate option. You cannot lie to the
parents and you do not know the likelihood of the child needing
a transfusion. You also cannot say for definite that this boy
won’t need one. If the surgery went ahead and he needed blood,
then the correct planning would not be in place and you would
be going against the parents’ (and child’s) wishes. This has
negative consequences for you as a clinician, and for the patient
and their relatives.

How well did you know this?
1
Not at all
2
3
4
5
Perfectly
2
Q
  1. You are working in a haematology clinic. You have been
    seeing Mrs Alcock regularly over the past 3 months. She
    says she would like to express her gratitude and hands you
    a £50 cheque. What should you do?
    Rank the following responses from most appropriate (=1) to
    least appropriate (=5):
    A. Accept the gift with thanks
    B. Politely decline the gift as it is too much
    C. Put the money towards a staff night out for everyone to enjoy
    D. Say it is hospital policy not to accept monetary gifts
    E. Record in the notes you accepted a monetary gift
A

ANSWER: DBECA
The GMC recommends that: “You must not encourage
patients to give, lend or bequeath money or gifts that will
directly or indirectly benefit you” (GMC 2008a).
D. Saying it is hospital policy not to accept monetary gifts is
the most appropriate option. Whether or not it is ‘officially’
hospital policy (in many trusts it is), you should decline the
patient’s gift in this manner as it shouldn’t cause too much
offence. If you simply decline the gift because it is too much
(B), then the patient might bring £20 next time. Under no circumstances
should you accept monetary gifts, as they could be
mistaken for a bribe.
B. Politely declining the gift as ‘it is too much’ is the next most
appropriate option. £50 is too much money to accept. Although
it is nice to receive gifts of gratitude from patients, you should
not be accepting large sums of money. Flowers and chocolates
as gifts are very different from money: you have to make a
sensible judgement on this.
E. Recording in the notes that you accepted a monetary gift is
the next most appropriate option. Although it is not appropriate
to accept such a generous gift, you should record it in the notes
if you did, and explain your reasons for accepting the gift.
C. Putting the money towards a staff night out is not appropriate.
Although, ideally, you should share gifts with your colleagues,
you cannot simply accept a monetary donation and
record it in another way. Usually patients will bring gifts for
the healthcare team, and so this issue is rarely a problem.
A. Accepting the gift with thanks is the least appropriate option.
This may encourage this patient to give further gifts and things
of monetary worth should not be accepted, as this could easily
be mistaken as bribery.

How well did you know this?
1
Not at all
2
3
4
5
Perfectly
3
Q
  1. You are taking blood from a 70-year-old woman on the
    medical ward who was admitted following a fall. While you
    are making conversation about her home circumstances, she
    tells you that she recently moved in with her son after her
    husband died. You enquire into how she is coping, and she
    becomes tearful, telling you that her son has started drinking
    and becomes very violent when intoxicated. This is the first
    time she has been in hospital for injuries caused by her son.
    Her son has assured her he will stop drinking so she doesn’t
    want to make a fuss. What should you do?
    Rank the following responses from most appropriate (=1) to
    least appropriate (=5):
    A. Inform the nurse in charge about the situation
    B. Find out whether she has anyone else she could stay with
    C. Contact your specialty trainee (registrar)
    D. Give her contact details of a local domestic support group
    E. Record details of your conversation in the notes
A

ANSWER: ACEBD
This question is asking you to consider the risk to this
elderly lady. An FY1 should always ensure the patient is the
focus of care.
A. Informing the nurse in charge about the situation is
the most appropriate option. This will directly deal with the
problem, as the nurses can refer to social services and take
the appropriate protective measures for this lady if her son tries
to visit her on the ward. Remember that nurses are the guardians
of safety and informing them about this lady is the best
thing you can do to protect her.
Answers 67
C. Contacting your specialty trainee is the next most appropriate,
as a more senior member of your team can give advice on how to
appropriately manage this situation. They should be informed of
those situations where patient safety is compromised.
E. Recording details of your conversation in the notes is the
next most suitable response as you need to record the patient’s
situation accurately. It is good practice to document everything
in the notes including the situation, who you informed and
actions taken. This covers you legally, hence the saying: if it’s
not written down, it never happened!
B. Finding out whether she has anyone else she can stay with
is the next most appropriate option. Although this finds out
more about her social circumstances, it does not formally
address the problem. Also, removing her from her current
social situation would not be your decision to make. Given her
age and fragility, this would require input from social services,
OT and physios about the suitability of a move.
D. Giving her the contact details of a local support group is
not appropriate. There has not been a decision made yet as to
how the medical team will best help this lady. Giving her details
of a local domestic support group may leave her feeling dismissed
as there is much more that can be done to help her. She
should not be made to feel as if she has to resolve anything
herself at this vulnerable time.

How well did you know this?
1
Not at all
2
3
4
5
Perfectly
4
Q
  1. A 14-year-old girl attends the GUM clinic asking for an
    STI check. Before you take some swabs, she tells you that
    she is pregnant. You confirm this with a pregnancy test. You
    ask her whether she has told anyone; she says no and that
    she especially doesn’t want her parents to know. What
    should you do?
    Rank the following responses from most appropriate (=1) to
    least appropriate (=5):
    A. Tell her that you have to let her parents know because she
    is under sixteen
    B. Try to persuade her it would be in her best interests to tell
    her parents
    C. Tell her that you will respect her wishes and not tell
    anyone
    D. Ask your consultant to see her
    E. Advise her that you are obliged to let her GP know
A

ANSWER: DBECA
This question is asking you to consider the health of the
unborn baby as well as the health of the teenager. Refer to
Chapter 2 for more information on under eighteens.
D. Asking your consultant to see her is the most appropriate
option, as you cannot assess Gillick competence as an FY1.
You should recognise that this is a difficult situation and you
need senior advice.
B. Trying to persuade her it would be in her best interests to
tell her parents is the next most appropriate option. Although
not every situation will warrant this action – for example, if it
was felt telling the parents could potentially harm the teenager
– you should have a sensible discussion about this. Ideally she
will be able to seek support from her parents.
E. Advising her that you are obliged to let her GP know is the
next most appropriate option, as you need to consider the health
of the unborn baby. This is where the GP steps in. Medical care
will be needed throughout the pregnancy, or for any other decisions
she may make. Informing her GP may also be a compromise,
if she refuses to let her parents know.
C. Telling her that you will respect her wishes and not tell
anyone is not appropriate. This would be dishonest, given
that you need to at least discuss it with your consultant and
the GP.
A. Telling her that you have to let her parents know because
she is under sixteen is the least appropriate response as you
should not break confidentiality without strong justification. If
she is Gillick competent, her wish for confidentiality should be
respected.

How well did you know this?
1
Not at all
2
3
4
5
Perfectly
5
Q
  1. The family of one of your patients who died is registering
    a complaint about their treatment. The solicitor calls you and
    asks for the medical records of the deceased to be sent to him.
    What should you do?
    Rank the following responses from most appropriate (=1) to
    least appropriate (=5):
    A. Send over the records as requested
    B. Say that you will get someone senior to call them back and
    take their details
    C. Remind them that any requests need to be sent in writing
    to the legal department
    D. Call your consultant and ask them to speak to the
    solicitor
    E. Take the solicitor’s details and tell them you will get the
    nurse to fax over the information
A

ANSWER: CDBEA
Remember that the duty of confidentiality persists after the
patient has died. Moreover, no party has a general right to
information, and therefore only information should be provided
that is relevant to the claim. No situation where there is a potential
breach of confidentiality should ever be rushed. Refer back
to Chapter 2 for more information on confidentiality.
C. Reminding them that any requests need to be sent in writing
to the legal department is the most appropriate option, and the
solicitor phoning should be aware of this. This directly – and
appropriately – deals with this problem. As an FY1 you should
not be involved in the sending of information to third parties,
but you can direct them to the appropriate department.
D. Calling your consultant is the next most appropriate
response, as they can more appropriately deal with this type of
request. They would also tell the solicitor to send it in writing
to the legal department.
B. Getting a senior to call them back is the next most appropriate
option. Options include contacting your F1 clinical lead,
foundation programme director or foundation programme coordinator
for legal matters. This action will delay tackling the
issue as compared with contacting your consultant.
E. Faxing over the information – even if a nurse does it – is
not appropriate. It is the legal department’s responsibility to
deal with this matter, not the medical team’s.
A. Sending over the records as requested is the least appropriate
response as this is a serious breach of confidentiality.

How well did you know this?
1
Not at all
2
3
4
5
Perfectly
6
Q
  1. You are in outpatients and you realise that your mobile
    phone has been stolen. You suspect one of eight patients
    could have taken it. You ring your insurance company and
    they tell you that you need to call the police and get a crime
    reference number. You call them, but cannot get a number
    without giving the police the names of the individuals in
    outpatients that day. What should you do?
    Rank the following responses from most appropriate (=1) to
    least appropriate (=5):
    A. Give over the names of the patients so that you can make a
    claim
    B. Fill in a critical incident (CAE) form
    C. Accuse patients you suspect and question them about it
    D. Ignore it and just replace your phone
    E. Report the incident to the foundation director
A

ANSWER: BEADC
This question requires recognition of the need to seek senior
advice in the case of a serious incident. For more information
on reporting concerns, refer back to Chapter 5.
68 Answers
B. Filling in a CAE form is the most appropriate response,
because it will put in place a system that should prevent future
thefts. Filling in the form needs to be done at the time of the
incident and not retrospectively.
E. Reporting the incident to the foundation director is the next
most appropriate option as this should help prevent future
recurrence. Your foundation director will also be able to offer
you advice on what you should do next. Remember, no matters
which involve breaking confidentiality are ever urgent.
A. Giving over the names of the patients so that you can make
a claim is the next most appropriate option. Although this does
involve disclosing patient information, it would be justifiable,
given the circumstances and having sought advice from a
senior. You are not expected to put up with having your phone
stolen
D. Ignoring the situation and replacing your phone is not
appropriate. You are not expected to put up with theft and
therefore this option – while in reality it may seem less complicated
– is not what you should do.
C. Accusing patients you suspect and questioning them about
it is the least appropriate response. This could produce potential
distrust between you and your patients creating a potentially
difficult situation for yourself.

How well did you know this?
1
Not at all
2
3
4
5
Perfectly
7
Q
  1. A 16-year-old girl comes into A&E with diabetic ketoacidosis
    (DKA). She is admitted and treated. The following
    morning you are arranging her discharge. You discuss her
    diabetic control with her. She says that she can’t be bothered
    with the insulin as it’s too much hassle. What should
    you do?
    Rank the following responses from most appropriate (=1) to
    least appropriate (=5):
    A. Educate her about the importance of compliance
    B. Arrange for the specialist diabetic nurse to see her before
    she goes
    C. Make a note of her non-compliance on the discharge
    summary to the GP
    D. Ask her parents to speak to her about compliance if they
    come to collect her
    E. Ask her why the insulin is such a problem
A

ANSWER: BEACD
This question is asking you to consider your opportunities
for learning. An FY1 should be willing to learn from others
and from their experiences.
B. Arranging for the diabetic specialist nurse to see her is the
most appropriate option, as they will have a wealth of experience
with non-compliance and may have some tricks up their
sleeve to deal with the situation. Ideally, you should utilise their
skills and expertise by sitting in on their consultation so the
nurse can educate you at the same time. This is the type of
experience you may choose to reflect on in your eportfolio
afterwards.
E. Asking her why the insulin is such a problem is the next
most appropriate option. Is it because she can’t do the same
things as her friends? Or maybe she has a problem with her
insulin regimen? Information means you can seek advice for a
more suitable treatment plan.
A. Educating her on the importance of compliance is the next
most appropriate, as it is your duty to inform patients of their
responsibility for their own health. You should use every available
opportunity you can for health promotion. The specialist
diabetic nurse, of course, would do this for you as part of their
consultation.
C. Making a note of her non-compliance on the discharge
summary to the GP is the next most appropriate option. The GP
should be made aware of the issue so that it can be followed up.
D. Asking her parents to speak to her about compliance is the
least appropriate option as – given her age – you should try to
engage with the patient first, rather than relying on her parents.
You should still speak to her parents though, as they may also
need educating about compliance. It would be best practice to let
the girl know that you would like to speak to her parents about
this. If there were circumstances in which she disagreed, you
should seek senior advice from your consultant or registrar.

How well did you know this?
1
Not at all
2
3
4
5
Perfectly
8
Q
  1. A 32-year-old woman presents to A&E for a head laceration.
    She claims that her boyfriend hit her, and she fell and
    hit her head. You smell alcohol on her breath. On questioning
    you find that she has left her two children at home under
    the supervision of her boyfriend who has also been drinking.
    You are worried about the children, what should
    you do?
    Rank the following responses from most appropriate (=1) to
    least appropriate (=5):
    A. Call social services to report your concern
    B. Call your registrar to assess the mother and discuss the case
    C. Fill in a referral form for social services
    D. Tell the mother it is protocol that you put in a referral to
    social services
    E. Tell the mother that you are concerned about the children,
    given her injuries
A

ANSWER: BEDAC
This question incorporates social services protocol and
expects you to demonstrate awareness of boundaries of
your own competence. An FY1 should readily seek help
when required. Refer back to Chapter 2 for specific information
regarding social services.
B. Calling your registrar to discuss the case is the most appropriate
option, as you will need senior consultation for advice
regarding social services. They will also be able to guide you
as to how to discuss the matter with the mother.
E. Telling the mother you are concerned about the children,
given her injuries is the next most appropriate option as you
first need to explain the reasoning behind your concerns
D. Telling the mother that it is protocol that you put in a referral
to social services is the next most appropriate option as you
need to explain what you are going to do about your concerns.
You should always try to inform the parent that you are putting
in a referral to social services, unless you feel that the child will
be put at risk if you do so. In either case you would bring in a
senior.
A. Calling social services is the next most appropriate option,
as they will be able to advise on the phone whether an urgent
response is necessary – i.e. to call the police to the house – or
whether a referral through the usual mechanism is necessary.
Social Services are the experts in this situation; hence, they
should be the ones dictating the action plan.
C. Filling in a referral form for social services is the least
appropriate option. Any form should be filled in at the time of
concern when all the relevant information can be collected;
but in this complex case it should be done by a senior. Moreover,
as there is an immediate concern, the phone call should
be made before the form is completed. However, if this was a
more “routine” case, then the phone call would not be necesAnswers
69
sary and you would simply consult with a senior about completing
the form.

How well did you know this?
1
Not at all
2
3
4
5
Perfectly
9
Q
  1. Mrs Smith is 53-year-old lady who suffered with heart
    failure for many years. She has expressed a wish to receive
    no further treatment and go home to rest in peace. The
    nurse comes to see you the following day, saying that her
    husband has arrived and is angry that you are sending her
    home to die. He feels his wife is “out of her mind and not
    thinking straight”. What should you do?
    Rank the following responses from most appropriate (=1) to
    least appropriate (=5):
    A. Go and see Mrs Smith to find out whether she remains firm
    in her wishes
    B. Ask the nurse to inform the husband it is Mrs Smith’s
    decision
    C. Explore the husband’s concerns
    D. Refer to psychiatry
    E. Persuade Mrs Smith to go with you to speak to her husband
    about her wishes
A

ANSWER: CAEBD
This question is about responding to a difficult situation
between a patient and their relatives and respecting the wishes
of both parties. An FY1 should understand both the relatives’
and the patient’s views for effective communication. See
Chapter 4 for more advice on effective communication.
C. Exploring the husband’s concerns is the most appropriate
thing to do. This is because it is important as far as possible to
respect the feelings of those close to the patient and meet their
needs for support.
A. Going to see Mrs Smith is the next appropriate option, as
you must find out whether her wish to refuse treatment is still
the same. Moreover, you must gain consent to discuss Mrs
Smith’s wishes with her husband.
E. Persuading Mrs Smith to go with you to speak to her
husband about her wishes follows on from A, as there is an
opportunity for conflict resolution if both parties communicate
their wishes to one another. This should be done once you have
obtained consent.
B. Asking the nurse to inform the husband it is Mrs Smith’s
decision is not really appropriate. While it is possible for team
members to resolve this issue, ideally it should be the person
with the best rapport. Nurses have a great deal of communication
with patients and their relatives and it is likely they’ve
approached you because they feel out of their depth. You should
appreciate the delicacy of this matter; simply directing the
nurse without exploring the issue further is not appropriate.
D. Referring Mrs Smith to psychiatry is the least appropriate
option as there is no indication that Mrs Smith needs psychiatric
assessment, nor of it being any benefit to her.

How well did you know this?
1
Not at all
2
3
4
5
Perfectly
10
Q
  1. Your FY1 colleague Mark has turned up late again for
    handover, and you smell alcohol on his breath. You know
    he has been having some family problems recently. You
    suspect the alcohol is from last night, but you cannot be
    sure. What should you do?
    Rank the following responses from most appropriate (=1) to
    least appropriate (=5):
    A. Report Mark immediately to the foundation programme
    director
    B. Ask your educational supervisor for advice
    C. Suggest to Mark that he explain the situation to the specialty
    trainee (registrar)
    D. Report the situation immediately to your consultant
    E. Have a quiet word with Mark after the handover to ask how
    he is coping
A

ANSWER: CDEBA
This question is about recognising and reporting dangerous
practice. An FY1 should challenge unacceptable behaviour
that threatens patient safety. Refer back to Chapter 5 for
more information on reporting concerns.
C. Suggesting to Mark that he explain the situation to the
registrar is the most appropriate option. This reduces the immediate
risk to patient safety as the registrar can suggest the
appropriate actions to take. This is an urgent situation as there
is a potential compromise for patient care. However, you might
not want to go straight to the consultant (D), as issues like this
will affect your colleague’s reputation. It is best if Mark can be
persuaded to report his irresponsible behaviour.
D. Reporting the situation immediately to your consultant is the
next most appropriate option. You have a responsibility to your
patients, and your duty of care to them overrides your loyalty to
your colleague. Serious incidents should immediately be raised,
if they compromise patient safety. You are not in a position to
deal with them yourself, so seniors must get involved.
E. Having a quiet word with Mark after the handover is the
next most appropriate option, as you should offer your colleague
the chance of an explanation and find out more about
the situation. It is important that you support your colleagues,
but having this conversation would not directly influence the
outcome of the scenario as you should address the issue of
patient safety first and foremost.
B. Asking your education supervisor for advice is the next
most appropriate option. Getting confidential advice on clinical
dilemmas should be considered, but this action will not address
the immediate problem. You should consult those in your
immediate team before more external clinicians. Although your
educational supervisor will be at your hospital, it may take time
to contact them.
A. Reporting Mark immediately to the foundation programme
director is not appropriate. Reporting procedure is well outlined,
and it is not your place to report this directly to the
foundation programme director. They would expect you to
go through the appropriate channels and try locally first. Moreover,
the foundation director may not be on site and may take
considerable time to respond which may result in a compromise
in patient care. Your foundation programme director would
eventually hear about it via the Doctors in Difficulty referral
pathway.

How well did you know this?
1
Not at all
2
3
4
5
Perfectly
11
Q
  1. An elderly woman is brought in by ambulance to A&E.
    She doesn’t speak English. The paramedics say that she was
    found collapsed on the street and a passer-by called 999.
    She is stable, but you suspect she needs to be kept in overnight.
    You name some languages and she nods at “Urdu”.
    It is 7.30 p.m., what should you do?
    Rank the following responses from most appropriate (=1) to
    least appropriate (=5):
    A. Arrange for an interpreter to come to the hospital
    B. Use an online translation service
    C. See if the patient can contact a friend or relative to
    interpret
    D. Arrange for a telephone interpretation
    E. Find a colleague on the ward and see if they can translate
A

ANSWER: EDABC
This question includes a common problem with translation
services and tests whether you can demonstrate initiative.
Refer back to Chapter 4 for more information on working
with interpreters.
E. Finding a colleague on the ward to see if they can translate
is the most appropriate option. Whilst it is not an ideal solution,
70 Answers
doctors and nurses remain impartial translators, and this means
that you can get any urgent information from the patient while
arranging for a more formal translator. Moreover, Urdu is a
common language, and the likelihood of finding someone in
the hospital who speaks it is high. Telephone services (e.g.
language line) are expensive and should be considered as the
next option if your colleagues cannot help.
D. Arranging for a telephone interpretation is the next appropriate
option, as most A&E departments should have access to
this service as arranged by the foundation trust. This service
allows access to interpreters anytime, day or night and offers
impartiality and confidentiality.
A. Arranging for an interpreter to come to the hospital is the
next most appropriate option. No matter where you are working,
the likelihood of getting an interpreter at 7.30 p.m. is slim. You
could, however, leave a message and arrange this for the following
morning. Typically, bilingual services are only available
from 9 a.m.–5 p.m.
B. Using an online translation service would require a computer
nearby that allows access to this service. It also requires that
you type in the information, which is time-consuming. Moreover,
there are limited languages available, and there is no guarantee
that the translation is accurate. These services are generally
inadequate for anything beyond the most basic of needs.
C. Seeing if the patient can contact a friend or relative to interpret
is not appropriate. The use of an ad hoc interpreter is discouraged,
and even more so if they have a close relationship with the patient.
This option should be avoided where at all possible.

How well did you know this?
1
Not at all
2
3
4
5
Perfectly
12
Q
  1. You are working on an oncology ward. It is a Saturday
    night and a patient is asking you for the results of his
    myeloma screen. The nurse tells you that the patient is
    going to complain if they do not get the results soon. The
    patient is anxious and cannot understand why it is taking
    so long. You do not have their results yet. What should you
    do?
    Rank the following responses from most appropriate (=1) to
    least appropriate (=5):
    A. Explain to the patient it is unlikely you will get the results
    before Monday
    B. Tell the nurse to go and explain to the patient that it is not
    possible to get the results until Monday
    C. Call the laboratory and put an urgent on the results
    D. Call the registrar and ask them to speak to the patient
    E. Apologise to the patient for the delay but explain that it is
    a weekend and unfortunately you will not have the results until
    Monday
A

ANSWER: EDACB
This question assesses your honesty and integrity with
regard to the patient relationship and ensures that you
make the patient your first concern.
E. Apologising to the patient for the delay is the most appropriate
response – and also the most honest. By apologising you are
showing empathy and acknowledging the patient’s distress.
D. Calling the registrar to speak to the patient is the next most
appropriate response, as this is a delicate issue that should be
dealt with by a senior colleague.
A. Explaining to the patient they are unlikely to get the results
before Monday is the next most appropriate response. This is
an honest answer, but does not acknowledge the patient’s concerns.
It would be more appropriate to have a senior review the
situation than to provide an inadequate explanation – especially
when there has been the threat of a complaint.
C. Calling the laboratory to put an urgent on the results is the
next most appropriate option, but this is unlikely to make any
immediate difference to the situation as it is the weekend and
other more urgent results will take priority.
B. Telling the nurse to explain the delay to the patient is not
appropriate. It is important that you go and see the patient
yourself and explain the situation. It is likely that the patient is
threatening to make a complaint because they are worried.
Hence, they will appreciate you taking the time to see them,
helping to ease their anxiety.

How well did you know this?
1
Not at all
2
3
4
5
Perfectly
13
Q
  1. You are working on a respiratory ward. A locum consultant
    prescribes antibiotics for Mr Jones for communityacquired
    pneumonia. You know these particular antibiotics
    are outside hospital guidelines. What should you do?
    Rank the following responses from most appropriate (=1) to
    least appropriate (=5):
    A. Prescribe the treatment as the consultant wishes but document
    in the notes they are outside of hospital guidelines
    B. Show the guidelines to the consultant and ask whether the
    antibiotics should still be prescribed
    C. Ask the consultant the reasons for prescribing those
    antibiotics
    D. Follow the guidelines and ignore the consultant’s prescription
    E. Ring pharmacy and find out their recommendations
A

ANSWER: BCEAD
This question analyses your consideration for hospital
policy and your ability to negotiate it with colleagues in an
appropriate manner.
B. Showing the guidelines to the consultant to ask whether
they should be prescribed is the most appropriate option as this
addresses the issue and considers the options for prescribing.
C. Asking the consultant the reasons for prescribing those
antibiotics is the next most appropriate option, because it may
be that the consultant is aware of the guidelines but has prescribed
them for a particular reason. Alternatively, these may
be the ones that he typically uses, in which case you could enter
into a discussion about local guidelines.
E. Ringing pharmacy to find out their recommendations is
more appropriate than A or D. Pharmacy are an excellent source
of guidance on the appropriate prescribing of medications. You
should consult expert advice rather than either brush aside your
concerns or undermine a colleague.
A. Prescribing the treatment the consultant wishes outside
guidelines without good reason is not appropriate, as you
should not ignore a situation where you think something is
wrong. However, it is more appropriate to follow your consultant’s
wishes than to undermine their expertise and not carry out
their management plan (D).
D. Following the guidelines and ignoring the consultant’s prescription
is the least appropriate option, as this means going
against the advice of the consultant. If the patient were to deteriorate
after you ignored their advice, you would be held soley
responsible.

How well did you know this?
1
Not at all
2
3
4
5
Perfectly
14
Q
  1. You are working a night shift on a surgical ward. A
    patient already on treatment for sepsis starts to rapidly
    deteriorate. It is nearly midnight. You complete your A
    to E primary survey and the patient is stable for the
    moment, but you feel you are out of your depth. You
    call the surgical registrar, but they tell you to call the
    medical team because they are busy in theatre. When you
    bleep the on-call medical registrar, they tell you it is not
    their responsibility and to find someone else. What should
    you do?
    Rank the following responses from most appropriate (=1) to
    least appropriate (=5):
    A. Call the surgical consultant at home
    B. Start basic management for the patient
    C. Call the surgical registrar again, explain the situation and
    ask for advice
    D. Ask a nurse to arrange for someone to help you
    E. Put out a periarrest call on 2222
A

ANSWER: CBDAE
Patient safety is always a first priority. It is important that
you recognise your limitations and seek appropriate support
when necessary. This should ideally be someone more experienced
than yourself.
C. Calling the surgical registrar again is the most appropriate
option, as they should give you the appropriate advice regarding
immediate management steps that can be taken.
B. Starting basic management for the patient is the next most
appropriate option, as you should take initial steps such as ordering
investigations, taking bloods, prescribing fluids etc. as part
of your secondary survey. These steps are within your remit, and
you should optimise the care you can give in this situation.
D. Asking a nurse to arrange for someone to help you is the
next most appropriate option as this involves sharing responsibility
for the patient. The nurses are perfectly equipped to seek
help for you while you attend to the patient. Moreover, they
will have knowledge of who else could be contacted in this
situation – for example, the hospital may have a critical care
outreach team. However, this is less appropriate than informing
seniors, as it does not directly address patient care. Moreover,
you have more knowledge of the patient’s clinical condition
and it is better to relay this information yourself.
A. Although calling the consultant at home is not a nice thing
to have to do, it is perfectly acceptable and would be the next
appropriate action if no seniors were available on site. They
will, however, be 20, 30 minutes away from the hospital
. . . maybe more. The patient, at the end of the day, comes
first and as the consultant’s responsibility overall, he/she would
rather be contacted regarding problems with patient care.
E. Putting out a periarrest call on 2222 is not appropriate, as
the question clearly says that the patient is stable. This should
only be used if you feel the patient is going to arrest. Think of
calling 2222 like dialling 999: for instance you would think twice
about calling if someone in a restaurant looked very well and
was simply having heartburn rather than having a heart attack.

How well did you know this?
1
Not at all
2
3
4
5
Perfectly
15
Q
  1. A patient with end-stage motor neurone disease asks
    you to give them the lethal injection. You explain to
    them that this is not legal in this country. What should
    you do?
    Rank the following responses from most appropriate (=1) to
    least appropriate (=5):
    A. Explore the patient’s reasons for wanting the lethal injection
    B. Tell the patient’s partner how they feel
    C. Ask the patient to talk to their partner about their feelings
    D. Refer the patient to psychiatry
    E. Explain to the patient about an advance directive and start
    the process
A

ANSWER: AECDB
This question explores patient focus. An FY1 is expected
to appreciate needs, build relationships with patients, be
respectful of patients’ wishes and work in partnership about
their care. However, FY1s are not expected to follow
patient’s wishes when it involves euthanasia!
A. Exploring the patient’s reasons for wanting the lethal injection
is the most appropriate option, because it is important that
you find out from the patient whether it is due to depression or
whether they are trying to convey specific preferences about
their treatment.
E. Explaining to the patient about an advance directive is the
next most appropriate option, as you would highlight the
choices available to them concerning their treatment.
C. Asking the patient to talk to their partner about their feelings
may result in them getting additional advice and support
if they so wish. This response, however, would not address the
immediate issue.
D. Referring the patient to psychiatry is not appropriate, as it
is not clear that the patient has a mental health issue. You would
also need senior input before making this decision.
B. Telling the patient’s partner how they feel is the least appropriate
option as you would be breaking patient confidentiality.

How well did you know this?
1
Not at all
2
3
4
5
Perfectly
16
Q
  1. An 83-year-old man is brought in to A&E after a fall.
    He has severe dementia and is obviously in pain, but is
    unable to tell you where the pain is coming from. After
    examination you suspect he has a broken hip. This is confirmed
    by X-ray. A decision needs to be made about treatment.
    However, he lacks capacity. His wife is deceased and
    his daughter, detailed as his next of kin, lives in Australia.
    What should you do?
    Rank the following responses from most appropriate (=1) to
    least appropriate (=5):
    A. Contact your local Independent Mental Capacity Advocate
    (IMCA) organisation
    B. Try to contact his daughter in Australia
    C. Speak to the registrar about scheduling him onto the surgical
    list
    D. Check through the notes for any legal documentation
    E. Start treatment anyway and give him analgesia
A

ANSWER: ECADB
This question tests your knowledge of consent where a
patient does not have capacity in an emergency situation.
Refer to Chapter 5 for more information.
E. Starting treatment is the most appropriate option, as the
patient comes first, and you do not need consent to give analgesia
– this is a basic treatment option that provides overall
benefit for a patient in pain.
C. Speaking to the registrar about scheduling him onto the
surgical list is the next most appropriate option. The responsibility
for decisions about treatment in an emergency situation lies
with the treating doctors. Emergency treatment must be provided
straight away. A senior needs to be made aware of the situation
ASAP as a broken hip can be potentially life-threatening.
Although it would not be your responsibility to schedule a patient
onto the surgical list, this option directly advocates for treatment
of the patient by directly involving their care.
A. Contacting your local IMCA is the next most appropriate
option. Typically, an IMCA is appointed in non-emergency
situations where a family member or friend cannot be contacted.
This service advocates for patients who lack capacity to
ensure that their feelings and wishes are considered. As broken
hips are scheduled onto the trauma list, this is an emergency
situation; however, as the patient has severe dementia, they will
be useful for assistance in less pressing matters. It is, therefore,
appropriate to contact them early in case a difficult decision has
to be made post-surgery.
D. Checking through the notes for any legal documentation is
the next most appropriate thing to do, as this will give you an
idea of the patient’s wishes as well as any documents detailing
72 Answers
a legal proxy whom you could contact to make his decisions
for him. Although this will be useful in informing decisions, a
thorough search should not prevent emergency treatment and
this could take some time given his age and co-morbidities.
B. Trying to contact his daughter in Australia is the least
appropriate option. Whilst you should contact a patient’s relatives
and close friends when trying to make a decision about
“overall benefit”, they do not have the final say in treatment.
You can assume that the patient would want them involved if
they lack capacity, but again this is different when the situation
is an emergency. This would not change your management of
the patient and is therefore the last action you would take in
this situation.

How well did you know this?
1
Not at all
2
3
4
5
Perfectly
17
Q
  1. It is a quiet afternoon on your surgical ward. The
    consultant asks you to come and assist with a private list
    in the afternoon at a different hospital. What should
    you do?
    Rank the following responses from most appropriate (=1) to
    least appropriate (=5):
    A. Ask your educational supervisor for advice
    B. Agree to help your consultant
    C. Decline and go down to the theatres to see if you can assist
    with any surgical cases
    D. Decline because you have responsibilities on the ward
    E. Ask your fellow FY1 colleague for advice
A

ANSWER: CDAEB
This question is about juggling your work commitments
with opportunities for career progression.
C. Declining and going to assist in theatres is the most appropriate
option, as you should utilise quiet times on the ward to
seek out additional learning opportunities. This option is appropriate
and enhances your educational development. Whilst it is
not appropriate to leave the hospital, it is perfectly acceptable
to seek experiences on site and, as a trainee, you should be
taking advantage of such situations.
D. Declining because you have responsibilities on the ward is
the next most appropriate option, as this immediately addresses
the issue. Whilst this would be a good opportunity, you should
not leave the hospital during paid working time. It would not be
fair to your colleagues to increase their workload. Nor would it
be fair to your patients whose care may suffer as a result. If you
explained this to the consultant, they should understand this.
A. Asking your education supervisor is the next most appropriate
option as you should ask for senior advice when faced with
a dilemma. Moreover, educational supervisors should be in
your hospital; therefore, you should receive a timely response
from them.
E. Should you not be able to contact your educational supervisor,
seeking advice from a fellow FY1 is the next most appropriate
response. Although they won’t have the same level of
expertise, they may have faced a similar situation, or know
someone else who has.
B. Agreeing to help your consultant with the private list is the
least appropriate option as it is unacceptable to leave without
formally handing over patients. Moreover, it is not appropriate
to leave your patients while you are being paid by the NHS to
care for them – no matter how quiet the ward is!

How well did you know this?
1
Not at all
2
3
4
5
Perfectly
18
Q
  1. You are seeing a patient in minors in A&E on a Friday
    night. A nurse comes in to tell you that a patient is being
    verbally aggressive and threatening because they haven’t
    been seen yet and thinks people are “jumping the queue”.
    The nurse suspects the patient has been drinking alcohol.
    What should you do?
    Rank the following responses from most appropriate (=1) to
    least appropriate (=5):
    A. Go and see the patient and tell them they will not be treated
    unless they calm down
    B. Ask the nurse to call security
    C. Tell the nurse to get someone else to see the patient because
    you are too busy
    D. Explain to the patient that you are very busy but that you
    will see them as soon as possible
    E. Physically restrain the patient
A

ANSWER: BDACE
Nurses handle a great deal on the wards. If they come to
see you about an aggressive patient, they will likely have
tried everything to calm them down, so be sympathetic.
B. Asking the nurse to call security is the most appropriate
option. Staff are not expected to tolerate abuse and calling
security is a necessary precaution as there is a potential for
this situation to get worse. Security can act quickly if it does
escalate.
D. Explaining to the patient you are busy but will see them as
soon as possible is the next most appropriate option, as patients
will often calm down if given a rational explanation.
A. Going to see the patient and telling them they will not be
treated unless they calm down is more appropriate than C and
E. However, a patient’s right to treatment should never be
influenced by their behaviour, and the GMC recommends that
you do not use treatment as a bargaining power.
C. Telling the nurse to get someone else to see the patient is
not appropriate. You might be the only person that the nurse
could find to help. If you did nothing and something were to
happen to the nurse because there was no one else around, then
you would be held accountable. This would be an error in
judgement. You should prioritise this situation as more urgent
than the patient you are seeing to in minors, as there is a potential
for harm.
E. Physically restraining the patient is the least appropriate
option. You cannot physically restrain a patient without just
cause, as restraint may be considered physical assault. This
would be the role of security.

How well did you know this?
1
Not at all
2
3
4
5
Perfectly
19
Q
  1. A 60-year-old Indian man comes into A&E with a
    history of fits. He was discharged 2 days ago from ITU. His
    wife is with him, but she does not speak English. He is very
    confused, disorientated and, from your A to E assessment,
    clearly unwell. You cannot get a history. It is 4 p.m. on a
    Monday afternoon. What should you do?
    Rank the following responses from most appropriate (=1) to
    least appropriate (=5):
    A. Proceed anyway with routine examination and investigations
    B. Ring the GP to get background information on the
    patient
    C. Ring the patient’s family to obtain the information
    D. Ask ITU to fax over their notes from the previous
    admission
    E. Arrange for an interpreter before you proceed
A

ANSWER: ADBEC
This question is about prioritisation and problem-solving. An
FY1 should demonstrate initiative when it comes to information
gathering and think creatively to solve problems.
A. Proceeding anyway with routine examination and investigations
is the most appropriate option as you must rule out
anything potentially life-threatening before you get more information
on this patient’s history. You can obtain a lot of information
from examination alone. It is not acceptable to neglect
examining a patient just because you can’t get a good history.
Patient care is your clinical priority and should always be done
Answers 73
first. If needs be, he can be treated as a new patient rather than
as a failed discharge.
D. Asking ITU to fax over the notes from the previous admission
is the next most appropriate option, as this will give you
access to the notes quickly – if they have them. They also might
have information on the patient’s family, translators, etc.
B. Ringing the GP to get background information is the next
most appropriate option. The GP should be the first port of call
for patient information. In this case, however, the GP may not
have the most recent information on the patient, given that it is
only two days since discharge and it typically takes longer for
discharge information to be received. Don’t be alarmed if the GP
asks to call you back on your bleep, or to fax the notes over – this
should be standard protocol to protect patient confidentiality.
E. Arranging for an interpreter is the next most appropriate
option. However, interpreters take time to arrange and it is
unlikely that you will get one within an hour. This situation is
potentially urgent and therefore other options should be
explored in preference to this.
C. Ringing the patient’s family is the least appropriate option.
This would break confidentiality and there is no guarantee that
they speak English either. His wife is already present with
whom the interpreter could consult.

How well did you know this?
1
Not at all
2
3
4
5
Perfectly
20
Q
  1. You are working as an FY1 on an obstetrics and gynaecology
    ward. A 29-year-old woman is admitted four hours
    into labour. This is her first pregnancy. She is requesting a
    Caesarean section (CS), but there is no medical indication
    for it at this stage. This is different from her birth plan, but
    the patient is demanding that she be given the care that she
    wants. What should you do?
    Rank these responses from most appropriate to (=1) to least
    appropriate (=5):
    A. Ask a midwife on the ward for advice
    B. Ring the consultant obstetrician to make them aware of the
    situation
    C. Explore the patient’s reasons for wanting a CS
    D. Explain the overall risks and benefits of both a CS and
    vaginal birth
    E. Refuse the CS as it is not medically indicated
A

ANSWER: CBDAE
This question is about working in partnership with patients
about their care whilst bearing in mind their best interests.
In terms of a patient’s best interests concerning a CS, indications
for a CS include (NICE 2011):
• Presumed foetal compromise
• ‘Failure to progress’ in labour
• Breech presentation (∼10% of all CS)
• Placenta praevia
• Multiple pregnancy
When a woman requests a CS NICE (2011) recommends:
• Discussing the overall risks and benefits of a CS and vaginal
birth, taking into account circumstances, concerns and priorities
• Including a discussion with other members of the obstetric team
(obstetrician, midwife and anaesthetist) to explore reasons for a
request and to ensure the woman has the accurate information
• Offering perinatal mental health support for women with
anxiety about childbirth
• If this is unsuccessful, offering a planned CS with a willing
obstetrician. If the consultant obstetrician is not willing, the
team should refer to one who is.
C. Exploring the patient’s reasons for wanting a CS is the most
appropriate option. This demonstrates consideration for their
needs and is the first step towards reaching a solution.
B. Ringing the consultant obstetrician is the next most appropriate
option as they will be making the final decision and need
to be made aware of the situation as soon as it arises. You will
also be able to discuss with them over the phone the risks and
benefits of both a CS and vaginal birth (D), which you could
relay to the patient before the consultant arrives.
D. Explaining the overall risks and benefits of a CS and vaginal
birth is the next most appropriate option. NICE (2011) recommends
this is carried out before any decision is made regarding
a CS. If the patient still wants to change her birth plan at
this point, then you have already appropriately informed your
seniors.
A. Asking the midwife on the ward for advice is the next most
appropriate option. The midwife is part of the obstetric team
and will undoubtedly have dealt with this situation before.
However, they would not make a final decision for this patient,
and therefore you should approach an obstetrician in preference
in this case.
E. Refusing the CS is the least appropriate option. This patient
has a right to a CS regardless of whether it is medically indicated,
and you are not in a position to refuse such treatment to
this patient.

How well did you know this?
1
Not at all
2
3
4
5
Perfectly
21
Q
  1. A 27-year-old woman comes into A&E with vomiting
    and mild abdominal pain. You have sent off bloods, but in
    the meantime the routine pregnancy test comes back positive.
    Her fiancé rings casualty to ask how she is doing. What
    should you do?
    Rank the following responses from most appropriate (=1) to
    least appropriate (=5):
    A. Tell her partner that she is pregnant
    B. Tell her partner that it is nothing serious and that she will
    be fine
    C. Tell her partner that he will have to speak to his fiancée
    directly
    D. Tell her partner that you will need to get her consent before
    you tell him anything
    E. Tell her partner that you are still waiting for the results of
    all the investigations
A

ANSWER: DCEBA
This question is about patient confidentiality. Refer to
Chapter 2 for more information on confidentiality breaches.
D. Telling her partner that you will need to get her consent
before you tell him anything is the most appropriate option, as
you must respect patient confidentiality. Generally, members of
the public understand this and will appreciate your honesty.
This option immediately resolves the issue.
C. Telling her partner that he will have to speak to his fiancée
directly is the next most appropriate response, as you are
respecting confidentiality whilst being honest with her fiancée.
E. Telling her partner that you are still waiting for the results
of all the investigations is the next most appropriate option, as
this does not involve giving the fiancé false reassurance (see
B). Although this woman is pregnant, there could also be other
explanations for her symptoms which you would want to
exclude before she was discharged. You are not being dishonest
with this explanation, as there are bloods pending.
B. Telling her partner that it is nothing serious and that
she will be fine is less appropriate, given that you cannot
74 Answers
confidently make this claim. This however is more appropriate
than A – actually breaking confidentiality.
A. Telling her partner that she is pregnant is not appropriate.
You do not have the patient’s consent to reveal this information,
and this would break patient confidentiality.

22
Q
  1. You are an FY1 working on the gastroenterology ward.
    A nurse comes to tell you that one of the patients with
    chronic alcoholism has been very rude. This is not the first
    time it has happened. The nurse is clearly quite upset, what
    should you do?
    Rank the following responses from most appropriate (=1) to
    least appropriate (=5):
    A. Tell the nurse you will speak to the patient
    B. Go and speak to the patient and tell them their behaviour is
    unacceptable
    C. Advise the nurse to ignore it
    D. Advise the nurse to avoid them and work in a different
    section
    E. Ask the patient to apologise to the nurse
A

ANSWER: ABEDC
This scenario involves prioritising your colleague’s needs
over the patients by offering them your support.
A. Telling the nurse you will speak to the patient is the most
appropriate option, as this shows your colleague that you take
their complaint seriously whilst giving the patient the opportunity
to express their own opinion on the situation. There are
two sides to every story and it is best to elicit them before
intervening.
B. Going to speak to the patient is the next most appropriate
option, as abuse towards the staff should not to be tolerated and
the patient should be made aware of this. This option also
allows you to find out the patient’s reasons for acting in the
manner the nurse described.
E. Asking the patient to apologise to the nurse is the next most
appropriate option. Out of respect to the nurse, it would be
polite to request that the patient apologise.
D. Advising the nurse to work in a different section is not
appropriate, as it does not directly address the problem. It will
be difficult for the nurse just to ignore part of the ward when
resources are already stretched. Moreover, this patient’s behaviour
may extend to abusing other members of the team, which
is unacceptable.
C. Advising the nurse to ignore it is the least appropriate option
as the question clearly states that this isn’t the first time it has
happened. Moreover, your colleague is upset; staff are not
expected to tolerate abuse in any form.

23
Q
  1. A patient – well known to psychiatry – is admitted onto
    the gastro ward with a history of somatisation disorder
    (multiple physical complaints with a psychological cause).
    She is demanding a bowel resection following a colonoscopy.
    The results of the colonoscopy are completely normal,
    and surgery is not clinically indicated. She is angry that you
    will not treat her and threatens to “cut it out herself if you
    don’t”. What should you do?
    Rank the following responses from most appropriate (=1) to
    least appropriate (=5):
    A. Agree with the patient that if she calms down you will
    consider surgery
    B. Bleep the on-call psychiatrist
    C. Reiterate to the patient that you cannot operate because the
    colonoscopy was normal
    D. Explain to the patient that, given the normal tests, you think
    this is part of their mental health problem
    E. Call the senior registrar for advice
A

ANSWER: DCEBA
This question is about responding appropriately to a difficult
patient with a mental health issue.
D. Explaining to the patient you think this is part of their
mental health problem is the most appropriate option. You are
explaining honestly the reasons for not operating. Moreover, it
may help calm the patient if you show that you have taken them
seriously, but that you are aware of their condition. You can
also assess the patient at this point to see how unwell they are.
C. Reiterating to the patient that you cannot operate because
the colonoscopy was normal is the next most appropriate
option. Repeating honestly why you cannot operate makes sure
you are consistent in your explanation.
E. Calling the senior registrar for advice is the next most
appropriate option. This is a difficult patient, and there is a
potential for serious harm should she act on her threats. You
should recognise that this is beyond your remit and seek help
accordingly. The registrar will also be able to advise you on
how to manage this patient further and on any other explanations
you can give to try and calm the patient down.
B. Bleeping the on-call psychiatrist is the next most appropriate
option. As the patient is already known to psychiatry, you should
seek their opinion, particularly given the serious threat of selfharm.
Following psychiatric assessment she may need admitting
to a psychiatric unit where she can receive alternative care.
A. Agreeing with the patient she can have surgery if she calms
down is the least appropriate option as under no circumstances
should you lie to a patient. This would have disastrous consequences
for the patient’s mental health: never bargain treatments
with patients.

24
Q
  1. You are working as an FY1 in a GUM clinic. You are
    seeing a young man with newly diagnosed Hepatitis B.
    He is an ex-intravenous drug user who works in a bar.
    You question him about his partner and discover that
    he has not disclosed his Hepatitis B status as he is afraid
    this will mean his partner will leave him. He says they
    are having protected sexual intercourse. What should
    you do?
    Rank the following responses from most appropriate (=1) to
    least appropriate (=5):
    A. Try to persuade him to tell his partner
    B. Do not disclose any information as there is no risk to his
    partner
    C. Explain you have a duty to tell his partner about their risk
    of infection
    D. Offer to talk to the patient and his partner together
    E. Recommend counselling
A

ANSWER: CADEB
This question requires you respect patient confidentiality.
In the case of communicable diseases the GMC recommends
(GMC 2009f):
• You should tell the patient that you will share their information
with those in the immediate healthcare team involved in
their care – unless they object.
• You cannot force a patient to reveal their infection status
unless someone is at risk of infection.
• You should explain to the patient how to minimise infection
risk to others.
• As with any issue, you can disclose information without
consent if justifiable e.g. serious harm.
• When tracing contacts, identity should not be revealed.
For more information on confidentiality see Chapter 2.
C. Explaining you have to duty to tell his partner about their
risk of infection is the most appropriate option, as this response
Answers 75
most directly considers the safety of his partner without putting
the patient in a more difficult situation. You have a legal
requirement to disclose the information to the person at risk
(GMC 2009f). It is also recommended that you tell the patient
before you make any disclosures and reiterate that they will not
be identified in the process.
A. Trying to persuade him to tell his partner is the next most
appropriate response, as the GMC (2009f) recommends that you
make every effort to persuade infectious patients to inform those
at risk. Given the hesitancy in this scenario, however, it is unlikely
that you will change his mind, and patients cannot be forced into
disclosing their diagnosis. It is therefore your responsibility rather
than the patient’s to ensure that information is shared.
D. Offering to talk to the patient and his partner together is
the next most appropriate option, as offering your support may
be all that is necessary for the patient to agree to disclosure.
This will relieve some of his pressure, as many patients find
breaking bad news extremely challenging. Lending an expert
opinion could make all the difference.
E. Recommending counselling is the next most appropriate
option. This should have been offered at time of diagnosis but
the scenario suggests they may be having difficulties coping
with the issue.
B. Not disclosing information because there is “no risk” is the
least appropriate option. As mentioned above, there is a potential
risk to the partner, and you have a duty to protect them, as
well as a duty to the patient.

25
Q
  1. You are an FY1 working on a labour ward. One of your
    patients has a breech presentation. She has a birth plan
    which specifies that every appropriate method should be
    attempted before opting for a Caesarean section (CS). She
    has now changed her mind and decided to have a CS. Her
    husband disagrees; taking you to one side and saying “she’s
    in pain, she doesn’t know what she wants, she’ll regret it if
    you operate”. What should you do?
    Rank the following responses from most appropriate (=1) to
    least appropriate (=5):
    A. Inform her of the overall risks and benefits of a CS and
    vaginal birth
    B. Call the consultant obstetrician to review the patient
    C. Enquire into her reasons for the change in birth plan
    D. Tell the husband you accept his point and remind him that
    it is his wife’s choice not his
    E. Ask the husband to leave so that you can speak to your
    patient in private
A

ANSWER: DCBAE
This question is about managing contrasting views of
patients and their relatives, necessitating good communication
skills. An FY1 is expected to adapt their style of communication
according to the context.
D. Reminding him that this is his wife’s choice is the most
appropriate option. You should respect the opinions of relatives,
but, at the end of the day, decisions about treatment should
always be made by the patient (provided they have capacity).
C. Enquiring into her reasons for the change in birth plan is
the next most appropriate option. You can then communicate
this to your consultant. It is best practice to get as much information
on the case before seeking senior advice.
B. Calling the consultant obstetrician to review the patient is
the next most appropriate option, as this is a decision above
your level; hence you need help. The situation is relatively
urgent given the potential risk to the mother and the unborn
child. The consultant will also be able to advise you on how to
best proceed with management in the interim.
A. Informing the patient about a CS and vaginal birth is
the next most appropriate option, as the patient needs to
be aware of the risks and benefits for both procedures. It is
likely that she will have been over this in her birth plan.
However, it is important to cover this again before any decisions
are made.
E. Asking the husband to leave so you can speak to the patient
in private is the least appropriate option. This removes the
support of her husband and could potentially cause more distress
to the patient. Also, the consultant would make a decision
whether the patient was being coerced by her husband or not.
You should try to maintain harmony in this scenario and removing
the husband is not the way forward.

26
Q
  1. You are working on a gastroenterology ward and a nurse
    approaches you and says Mr Brown needs to be prescribed his
    usual fluids. You are finishing up a discharge summary and
    take home drugs for another patient. What should you do?
    Rank the following responses from most appropriate (=1) to
    least appropriate (=5):
    A. Prescribe Mr Brown normal saline
    B. Go and see Mr Brown straight away
    C. Explain that, to enable you to get the discharges done on
    time, they should ask another FY1 to help
    D. Say you will put it on your jobs list and get to it as soon as
    you can
    E. Ask about Mr Brown’s fluid status
A

ANSWER: EDCBA
This question requires you to consider prescribing errors and
patient safety in a pressured environment. An FY1 is expected
to remain calm when under pressure, demonstrate good judgement
and manage uncertainty. Refer to Chapter 3 for more
information on safe prescribing and prioritisation.
E. Asking about Mr Brown’s fluid status is the most appropriate
option. You need to find out more information from the
nurse about Mr Brown before you can make an educated decision
about the urgency of this task. For example, if the nurse
was concerned about a low urine output, you might go and see
him straight away. If there was no concern, then you would
finish the prescription and see him afterwards. You should
always prioritise tasks according to clinical need.
D. To say you will put it on your jobs list and get to it as soon
as you can is the next most appropriate option as this means
you can prioritise it accordingly. Discharge summaries are
important and involve prescribing and dosages. You should not
be distracted from this task if possible as it may mean that the
patient is discharged without the appropriate medications, or
on the wrong medication etc.
C. Explaining that to enable you to get the discharges done
they should ask another FY1 is an abrupt response, but prioritising
the discharge summaries over this routine prescription is
more appropriate than (B) or (A). You should be supportive
towards colleagues, but recognise when to say no. Moreover,
the nurses and ward clerks are under pressure to discharge
patients and they should understand your reasons.
B. Going to see Mr Brown straight away is a less appropriate
option. In this case, the priority of care is ensuring that the
76 Answers
patient you are discharging gets the right medication and has
the relevant information communicated to their GP. There is no
indication in this scenario that Mr Brown is unwell and therefore
his “usual” fluids can wait.
A. Prescribing Mr Brown his normal saline is the least appropriate
option, as this is potentially dangerous. If you overload
Mr Brown with fluid you could cause pulmonary oedema.
Always remember to see the patient before you prescribe in
order to minimise error.

27
Q
  1. A colleague has left 30 minutes early to attend a dental
    appointment. They have asked you to prescribe Warfarin
    for their patient. Your shift has already ended, and you were
    about to go home. The INR result from the morning’s blood
    tests is not back yet. What should you do?
    Rank the following responses from most appropriate (=1) to
    least appropriate (=5):
    A. Prescribe the Warfarin anyway
    B. Hand over the request to the ward cover
    C. Write it onto your jobs list for the morning
    D. Write that the patient needs their Warfarin dosing in their
    notes
    E. Ring the lab to see if the INR results are there
A

ANSWER: EBDCA
This question is about safe prescribing and prioritising
patient care. Refer to Chapter 3 for information on pressures,
prescribing and prioritisation.
E. Ringing the lab to see if the INR results are available is the
most appropriate option, as you should find out why the results
have taken so long. It may be that the sample has been missed
off, in which case you would need to rebleed the patient – or
ask them if they can do a repeat. Usually the lab will save part
of the sample for such cases. Whilst you are not expected to
stay long after your shift has ended, you have accepted responsibility
for this patient and therefore you should make their care
a priority before you leave.
B. Handing over the request to the ward cover is the next most
appropriate option, as you should make sure that the patient has a
continuation of care and receives their Warfarin in a safe manner.
D. Recording that the patient needs their Warfarin dosing in
the notes is the next most appropriate option. Although writing
in the notes does not ensure that it will get done, having a record
means you can check it in the morning. Not having the patient’s
Warfarin dosed could be equally as detrimental as overprescribing
it. It is always good practice to write in the notes as well
as make a personal list.
C. Writing it onto your job list for the morning is the next most
appropriate option, as you should make a note to yourself to
check that this has been done. After all, this patient is now your
responsibility.
A. Prescribing the Warfarin anyway is not appropriate, as prescribing
this without the INR result potentially puts the patient
at risk of bleeding.

28
Q
  1. You are working as an FY1 on a surgical ward. On your
    way to ordering a CT scan from radiology, you are bleeped.
    You find the nearest phone to ring through. A nurse is concerned
    about Mrs Fazi’s urine output (UO). You remember
    it was fine for her small frame when you checked two hours
    ago. What should you do?
    Rank the following responses from most appropriate (=1) to
    least appropriate (=5):
    A. Ask the nurse to do some basic observations on Mrs
    Fazi and say you will come as soon as you have ordered
    the CT scan
    B. Go and see Mrs Fazi straight away
    C. Reassure the nurse that Mrs Fazi’s UO was fine for her size
    two hours ago
    D. Ask the nurse whether Mrs Fazi’s UO has changed since
    this morning
    E. Tell the nurse to start 1 litre of normal saline over 8 hours
    for Mrs Fazi, and you will prescribe it once you have been to
    radiology
A

ANSWER: DACBE
This question is about coping with pressure. An FY1 is
expected to adapt to changing circumstances and manage
uncertainty. They should also be able to re-prioritise tasks
as necessary.
D. Asking the nurse whether Mrs Fazi’s UO has changed since
this morning is the most appropriate option as you need to find
out more about Mrs Fazi before you can make a decision.
A. Asking the nurse to do some basic observations whilst you
order the CT is the next most appropriate option as you need
to get scans approved early in the day for them to get done.
The scan is, therefore, prioritised as urgent. Not completing this
task may cause that patient to suffer. Your consultant will not
be impressed if you don’t get the scan, as it will mess up the
patient’s management plan.
C. Reassuring the nurse that Mrs Fazi’s UO was fine for her
size is the next most appropriate option, as it may be that the
nurse hasn’t accounted for her small frame. If it was fine two
hours ago, you can reassure the nurse that it is sensible to order
the CT scan first before going to check on Mrs Fazi.
B. Going to see Mrs Fazi immediately is not appropriate. You
should not drop everything when there is no indication that
there is an emergency. You should address the nurse’s concerns
in a timely manner, whilst making sure that you prioritise
effectively. It is better to spend 15 minutes ordering a CT scan
and attend to Mrs Fazi after.
E. Telling the nurse to start fluids is not appropriate, as you
should not prescribe over the phone, nor should you prescribe
for a patient that you haven’t been to see yourself.

29
Q
  1. You go to see a patient with whom you previously had
    trouble putting in a cannula. You ask them whether you can
    take some blood. They recognise you and say “don’t come
    anywhere near me with that thing, you don’t know what
    you’re doing”. What should you do?
    Rank the following responses from most appropriate (=1) to
    least appropriate (=5):
    A. Reassure the patient that you are competent at taking blood
    B. Go and ask a nurse to take their blood
    C. Apologise to the patient for yesterday
    D. Tell the patient you have to take their blood as there is no
    one else
    E. Take the blood from their cannula
A

ANSWER: ACBDE
This question is about taking responsibility for your own
actions and owning up to mistakes.
A. Reassuring the patient that you are competent at taking
blood is the most appropriate option as you should reassure the
patient of your clinical skills. This response directly addresses
the situation.
C. Apologising to the patient for yesterday is the next most
appropriate option, as you need to show the patient that you
accept responsibility for causing them distress. The patient may
also be more likely to consent to venepuncture on acceptance
of your apology. This, however, would be best after some reassurance
of your ability.
B. Asking a nurse to take their blood is the next most appropriate
option, as you should not lie to force the patient to consent
(D) and you cannot take blood from the cannula (E) as it will
Answers 77
be contaminated. Although nurses are busy, they can always
help you out and some will be trained in taking blood.
D. Telling the patient that you have to take their blood because
there is no-one else is not appropriate. This is coercing the
patient to consent. You should attempt to find someone else to
take it before you tell them that they have no other options.
E. Taking blood from their cannula is not appropriate. There
are few occasions when you can take blood from a cannula as
fluids and drugs will have passed through it. The sample would
be contaminated, which means the lab would process the bloods
erroneously.

30
Q
  1. A patient you recently discharged from the ward
    requests your friendship on a social networking site. They
    are a similar age to you. What should you do?
    Rank the following responses from most appropriate (=1) to
    least appropriate (=5):
    A. Accept the friend request but do not engage with the former
    patient
    B. Decline the friend request and explain your reasons
    C. Ignore their friend request
    D. Ask your FY1 colleagues for advice
    E. Ask your educational supervisor for advice
A

ANSWER: BECDA
This question is about professionalism. For advice on social
networking refer to Chapter 2.
B. Declining the friend request and explaining your reasons is
the most appropriate option. The BMA (2011) recommends that
you decline any offers of friendship and explain why to the
patient. This most effectively addresses the problem.
E. Asking your educational supervisor for advice is the next
most appropriate option, as they will be able to explain what
to do in this situation and turn it into a learning experience. You
gain more from this than you would from ignoring the situation
and not seeking advice.
C. Ignoring the request is the next most appropriate option, as
it is better to ignore a friend request than accept it. This would
address the problem but not necessarily solve it. It would be
more appropriate to learn from the experience by seeking
advice.
D. Asking your FY1 colleague for advice is the next most
appropriate option. Although this will not directly solve the
issue, your colleagues should have an understanding of professionalism
and best practice and may be able to give advice.
A. Accepting the request is not appropriate. It is not advisable
to accept online friendships with patients or former patients.
Irrespective of their discharge, they could at any point come
under your care again, and it is unprofessional to enter into this
kind of relationship.

31
Q
  1. You are working on ITU, completing the morning’s job
    list. When taking bloods from a patient – admitted following
    a road traffic accident – you accidently stick yourself with
    the needle. You had followed infection control guidelines and
    were wearing gloves, but you see your finger bleeding underneath.
    You quickly squeeze it, clean it and put a dressing on
    it. You go to look on the system for anything on the patient’s
    HIV/Hepatitis status but there is nothing. The patient is
    unconscious. What should you do?
    Choose the three most appropriate options from the following
    list:
    A. Request viral serology on the blood forms for HIV and
    Hepatitis status
    B. Ring occupational health
    C. Go down to A&E to get some anti-retrovirals to start
    immediately
    D. Fill in a CAE form for the stick injury
    E. Ask if you can go home because you are so worried
    F. Wait to see if the patient wakes up and ask their permission
    for serology
    G. Bleep your registrar to explain the situation
    H. Explain to one of the nurses what has happened
A

ANSWER: BDG
This question is asking whether you understand best practice
following a stick injury.
B. Ringing occupational health is appropriate, as they will be
able to tell you immediately the protocol for stick injuries.
D. Filling in a CAE form is appropriate, as when you go to occupational
health, they will ask whether you have filled this in.
G. Bleeping your registrar to explain the situation is appropriate,
as you will have to go to occupational health, thereby
leaving ITU. You should always notify a senior of a serious
incident – as in this case – so they can make the appropriate
arrangements such that neither your own nor the patient’s safety
is compromised.
A. Requesting viral serology is not appropriate, as the patient
has not consented to having these investigations and it would
not be in their best interests to have them done. If they were in
ITU for an illness in which HIV status could benefit their treatment,
then this could be considered. You, however, would not
request this; it would have to be done by a senior.
C. Going to A&E for anti-retrovirals is not appropriate during
working hours. Occupational health will be able to give you
the appropriate advice and treatment. If occupational health
cannot be contacted, you should go to A&E where an appropriate
risk assessment and management can be carried out.
E. Asking if you can go home is not appropriate, as this would
be a way of avoiding, rather than helping, this situation.
F. Waiting to see if the patient wakes up to ask for their permission
for serology is not appropriate. The patient is unconscious
and therefore you have no idea when they will wake up.
You should act quickly and promptly after a stick injury to
reduce the risk to yourself as far as possible.
H. Explaining to one of the nurses what has happened is not
a preferred option as you should inform your senior before the
nurses. The nurse’s priority is largely patient safety, whereas a
senior colleague will be able to analyse the situation and
manage the team to best support your needs and ensure the
patient’s best interests are considered.

32
Q
  1. Your FY1 colleague turns up late again for their shift.
    The ward you are working on is quiet. Your colleague
    arrives in tears; she is feeling exhausted, stressed and
    says that she is not coping with anything at the minute.
    This is the third time in the past two weeks. What should
    you do?
    Choose the three most appropriate options from the following
    list:
    A. Apologise and say that you are busy now but that you will
    talk to her after your shift
    B. Sit her down to have a talk about her problems
    C. Suggest she books an appointment to see her GP
    D. Suggest she speaks to her registrar
    E. Suggest she takes sick leave
    F. Advise her to talk to her educational supervisor
    G. Seek advice about the situation from your educational
    supervisor
    H. Mention that you have to inform your consultant of her
    difficulties
A

ANSWER: BFC
This is about supporting a doctor in difficulty and prioritising
your colleagues.
B. Sitting your colleague down to talk about her problems is
the most appropriate option. Your colleague is clearly distressed
and therefore you should deal with this issue in a timely manner.
F. Advising your colleague to talk to her educational supervisor
is also appropriate, as their supervisor should be the “go-to”
78 Answers
person for support. They have access to the “doctors in difficulty”
programme, if needed. It is always better that you flag
up issues with your supervisor rather than ignoring them and
letting them be referred in via a different avenue.
C. Suggesting she book an appointment to see her GP is an
appropriate option. Although you don’t yet know what is
causing her stress, it might be good for her to talk to someone
external without the added pressure of feeling judged by her
seniors. GPs are also used to seeing patients with health problems
caused by stress on a daily basis and will be able to recommend
a sensible way forward – including recommending
sick leave.
G. Seeking advice from your educational supervisor is not a
preferred option. Although your educational supervisor is a
good point of contact when you are unsure how to handle a
situation, your supervisor can’t do much to help your colleague
if she has a different educational supervisor. Your educational
supervisor could be helpful if you needed to debrief on how
you handled the situation.
E. Suggesting she take sick leave will not address the issue and
is therefore not appropriate. If it involves stress at work, taking
leave will only prolong it until they return. Also her GP would
be the most appropriate person to make this decision.
A. Apologising and saying that you are busy is not appropriate,
given how upset your colleague is, and the question clearly
states that you are on a quiet ward. Moreover, this isn’t the first
time she has displayed a significant amount of distress.
D. Suggesting your colleague speak to her registrar is not a
preferred option. While she could speak to her registrar, it is
sensible to seek advice from those in the best position to help.
H. Mentioning that you have to inform your consultant of
her difficulties is not appropriate, as this may alienate your
colleague further. As there is no indication of compromise
to patient safety, you should be supportive rather than accusatory
and recommend she seek help through the appropriate
channels.

33
Q
  1. You are working in Children’s A&E on a Saturday
    night. A 13-year-old boy comes in drunk. He admits he has
    been drinking alcohol and says he fell over. The X-ray
    confirms he has broken his index and middle fingers. The
    boy is adamant he is fine and doesn’t want any treatment
    because he is scared that his parents will find out he has
    been drinking. He wants you to just give him some painkillers
    and then he’ll leave. What should you do?
    Choose the top three most appropriate options from the following
    list:
    A. Prescribe the painkillers and let him self-discharge
    B. Ring the boy’s parents without telling him
    C. Explain that you really need to let his parents know what
    has happened and it would be best if he rang them
    D. Tell him to go and see his GP if he has any problems over
    the weekend
    E. Try to persuade him to have his fingers splinted
    F. Put in a referral for social services
    G. Refuse to let him self-discharge without getting the proper
    treatment
    H. Discuss the situation with the nurse in charge
A

ANSWER: HEC
This question is asking you about acting in the patient’s best
interests for someone who does not have the capacity to
refuse treatment. See Chapter 2 for more information on
patient advocacy for the under eighteens.
H. Discussing the situation with the nurse in charge is appropriate,
as the nurses on this unit will have experience in dealing
with teenagers refusing treatment. They will be able to give you
advice. They also might be able to talk to the boy and get him
to stay.
E. Trying to persuade him to have his fingers splinted is appropriate,
as you should make every effort to help this boy understand
that he should have treatment. This is easier than having
to force him to stay.
C. Explaining that you really need to let his parents know what
has happened and it would be best if he rang them is appropriate,
as this boy is a minor and has been drinking alcohol, and
therefore is unlikely to be Gillick competent. He is also at risk
of harm from further alcohol abuse and injury. If he self-discharged
and his parents knew nothing about it, your professionalism
could be questioned.
A. Prescribing the painkillers and letting him self-discharge is
not appropriate. You should not prescribe pain relief if he has
been drinking alcohol; moreover, it is in his best interests that
he does not self-discharge.
B. Ringing the boy’s parents without telling him is not appropriate,
as this is breaking confidentiality. You should always
inform the patient if you are going to do this.
D. Telling him to go and see his GP if he has any problems
over the weekend is not appropriate, as he has not self-discharged
yet.
F. Putting in a referral for social services is not appropriate
at this stage as there is no need for intervention by social services
on this evidence alone.
G. Refusing to let him self-discharge without getting the
proper treatment is not a preferred option. A teenager can only
consent to treatment; they cannot refuse treatment even if
they are Gillick competent, as this must be overridden if
you consider treatment to be in their best interests. It is, therefore,
more appropriate to get senior support. Remember to
select the options that are the most appropriate to your role
as an FY1.

34
Q
  1. You are on a ward round with your consultant on the
    coronary care unit. The patient’s relatives are sitting in the
    waiting room at the end of the corridor. Your consultant
    says that the patient is going to die soon and checks the Do
    Not Attempt Resuscitation (DNAR) form. The family
    approach you politely after the ward round whilst you are
    ordering bloods and say “so you think Mum is going to die,
    do you?” It is clear they have overheard your consultant’s
    comments. What should you do?
    Choose the three most appropriate options from the following
    list:
    A. Ask a nurse to take the family to a side room and explain
    you will follow shortly
    B. Ask them where they heard that information
    C. Tell them you are busy at the moment and will talk to them
    later
    D. Apologise to the family for the fact that they overheard the
    conversation with your consultant
    E. Fill in a critical incident (CAE) form
    F. Ask the family if they would like to sit with their mum
    G. Finish ordering the bloods
    H. Call your consultant, explain what happened and ask them
    to speak to the family
A

ANSWER: HAD
This question considers how you deal with a difficult situation
involving relatives. Remember that good communication
is TPP: at the right Time, with the appropriate People
in the appropriate Place.
H. Calling your consultant to explain what has happened is
appropriate. This is a complex situation and your consultant
Answers 79
really is the most appropriate person to deal with this matter.
You can discuss with the consultant over the phone how you
should proceed.
A. Asking a nurse to take the family to a side room is appropriate,
as this is a sensitive issue that would best be approached
in a neutral space. Moreover, as it is a potentially volatile situation,
having a colleague for support is a good idea.
D. Apologising to the family is appropriate, as you will clearly
have caused the relatives some distress in hearing about their
mother in such frank terms. An apology demonstrates your
sensitivity towards their needs.
F. Asking the family if they would like to sit with their mother
is not a preferred option, given the nature of the situation. They
should be directed to an appropriate room to await the consultant
to have a detailed conversation. They may wish to sit with
their relative after this time, but it would be best to have a
discussion on neutral ground rather than at the bedside.
B. Asking where they heard that information is not appropriate,
as it is obvious there has been error, irrespective of where
they heard this information.
C. Telling them you are busy and G. Finishing ordering the
bloods are not appropriate, as both actions are rude and do not
appropriately deal with this situation. Bloods can wait for 20
minutes whilst this issue is addressed.
E. Filling in a CAE form is not a preferred option. This could
be considered at a later stage after the incident has been
managed appropriately. Also, the consultant may wish to complete
the form in this case.

35
Q
  1. A 9-year-old boy was admitted following an exacerbation
    of asthma. You speak with the parents about using his
    steroid inhaler. The boy’s mother is happy to make sure he
    has the inhaler; however, his father is clearly not. Father
    says that he has heard that they will stunt his boy’s growth
    and he doesn’t want that. What should you do?
    Choose the three most appropriate options from the following
    list:
    A. Ignore the father’s wishes and prescribe the inhaler anyway
    as the mother has consented
    B. Contact social services about the disagreement
    C. Ask the asthma specialist nurse for advice
    D. Agree with the father not to use the steroid inhaler
    E. Get the mother to persuade the father the steroid inhaler is
    necessary
    F. Plan to monitor the boy’s growth on a growth chart and
    review throughout the use of his inhaler
    G. Educate the father about the advantages to using the
    inhaler
    H. Ask the boy what he would like to do
A

ANSWER: GFC
This question is about acknowledging a parent’s concerns
and managing them appropriately to ensure compliance
with medication which is in the best interests of your patient.
G. Educating the father about the advantages of using an
inhaler is appropriate, as the father may have some misconceptions
– in which case you can iron them out through education
and help change his mind about the inhaler.
F. Planning to monitor the boy’s growth on a growth chart is
also appropriate, as this is addresses the concerns of the father
by diligently monitoring for any side effects. By recording the
boy’s growth on the chart, you are showing that you take his
misgivings seriously, which will hopefully improve compliance
in the long term.
C. Asking the asthma specialist nurse for advice is another
appropriate option, as it is likely the specialist nurse will have
dealt with a similar scenario before and may have some tricks
to help handle the situation.
H. Asking the boy what he would like to do is not a preferred
choice as although you should try to involve any child as much
as possible in their care, acknowledging the parent’s disagreement
should be the priority. Moreover, this boy is only nine
years old and too young to consent to treatment himself.
E. Getting the mother to persuade the father is not a preferred
option as you must try to educate the father first, and get some
more advice, before using the other parent as the mediator. The
evidence of healthcare professionals should be much more
effective than the mother’s opinion alone. Moreover, this would
not adequately address the father’s concerns.
B. Contacting social services about the disagreement is not
appropriate, as there is nothing to indicate that this child is at
risk of harm.
D. Agreeing with the father not to use the steroid inhaler is not
appropriate, as it would not be in the best interests of the child
to stop his asthma medication.
A. Ignoring the father’s wishes is not appropriate, as you
should make every effort to resolve the dispute so that the child
receives the same message from both parents. If they conflict,
mixed messages can lead to increased non-compliance. Be
careful not confuse the issue of consent with the issue of compliance.
Although only one parent is needed to consent to treatment,
in this case you want both parents on the same page so
that the boy receives his medication in the appropriate manner.

36
Q
  1. A 43-year-old man is brought in by ambulance to A&E
    after collapsing in a bar on a Friday night. You have no
    details other than his name and date of birth, which the
    paramedics got from his wallet. The man is incapable of
    giving a history due to his level of intoxication. He is stable
    in the resuscitation area. What should you do?
    Choose the three most appropriate options from the following
    list:
    A. Continue with your A to E assessment
    B. Ring your specialty registrar
    C. Ask a clerk to search the hospital database for his records
    D. Search through his mobile phone contacts to find a family
    number and call them
    E. Start treatment anyway
    F. Call the cardiac on-call SHO
    G. Start the appropriate investigations but do not start any new
    medications
    H. Ask one of the nurses to try and track down some information
    on him
A

ANSWER: AGC
This is about patient prioritisation in an emergency situation
and recognising your job role and limitations. An FY1
should be proactive and able to initiate assessment and
investigations but know when to seek help: in this case for
starting treatment.
A. Continuing with your A to E assessment is appropriate, as
you should always go back to basics for every patient.
G. Starting the appropriate investigations but no new medications
is appropriate, as you must treat the patient as new if you
don’t know anything. You can hold off starting any medications
until you have tried to find out whether he has any allergies or
is on any medications that may interact.
C. Asking a clerk to search the hospital database for any
records is appropriate. Patient care should be your priority, and
finding more about his medications and past medical history is
essential. A clerk should have time to look through the database
and print off any old clinical letters to find out more about him.
80 Answers
B. Ringing your registrar is not a preferred option. You should
complete your A to E assessment and then see what you can
find out about his history. It says in the question that this man
is stable; therefore you can assume that it is better to find out
as much as you can before calling for help in this case. If the
scenario was an emergency, however, you would call for help
immediately.
D. Searching through his mobile phone is not an appropriate
option, as you should not be using friends and family to get
information about a patient unless it is a last resort. You should
check the hospital database first.
E. Starting the treatment anyway is not a preferred option, as
you should start investigations before management, G is therefore
more suitable.
H. Asking one of the nurses to try and track down some information
is not appropriate. Nurses are busy, and you are expected
to take responsibility for finding patient information – or delegate
it to the ward clerk if you are busy with the patient.

37
Q
  1. A 24-year-old woman is admitted to A&E following a
    paracetamol overdose. Her boyfriend brought her in when
    she admitted that she had taken 50 × 500 mg paracetamol
    tablets one hour earlier. Her boyfriend tells you she has
    been suffering from depression. You speak with her alone
    and she refuses any form of treatment – she wants to die.
    What should you do?
    Choose the three most appropriate options from the following
    list:
    A. Respect her right to refuse treatment
    B. See if her boyfriend can persuade her to change her mind
    C. Bleep the on-call psychiatrist
    D. Call your registrar
    E. Treat her under the Mental Health Act
    F. Treat her under the Mental Capacity Act
    G. Treat her under the doctrine of necessity
    H. Discharge her from A&E
A

ANSWER: CBD
This is about recognising the need for senior help and tests
your ability to handle patients who refuse to consent to
treatment. Refer to Chapter 4 for more information on difficulties
consenting patients.
C. Bleeping the on-call psychiatrist is appropriate, as you have
information from her boyfriend that she has been suffering from
depression. Moreover, this should be treated as an attempted
suicide and therefore warrants psychiatric assessment. If you
spoke to your registrar, they would advise you on a psychiatric
referral. In this case you would need a rapid assessment on
whether she has the capacity to refuse treatment.
B. Seeing if the boyfriend can persuade her to change her
mind is appropriate. Using family and friends can be extremely
helpful in getting patients to comply with treatment. Although
this also puts him in a difficult situation, she may find it helpful
talking to someone that she knows and has a close relationship
with. You would still need to speak to psychiatry however.
D. Calling your registrar is appropriate, as this is not something
that you would be expected to handle on your own as an
FY1. You should call for senior help as soon as possible as this
is a challenging situation.
E. Treating her under the Mental Health Act (MHA) is not
a top selection, but is more appropriate than options F –
Treating her under the Mental Capacity Act (MCA) – and G –
Treating her under the doctrine of necessity – but you would
need a full assessment before jumping to commit her to treatment
without consent. You would choose E over F as she doesn’t meet
the criteria for the MCA in that she understands that not having
treatment will kill her. However, she is suffering from a mental
health disorder and therefore can be treated against her will under
the MHA. The situation of urgency is not so great that the doctrine
of necessity (G) is warranted. Although this is an emergency,
it is not such that you override the patient’s wishes and
act in their best interests anyway. She needs further assessment
of her competence to refuse treatment.
A. Respecting her right to refuse treatment is not appropriate,
as the history suggests that this patient’s judgement may be
compromised
and therefore you should explore other options
first.
H. Discharging her from A&E is not appropriate. You have a
duty of care to this patient and dismissing her would question
your professionalism.

38
Q
  1. You are working on a labour ward as an FY1. A nurse
    comes to see you to let you know that the ex-partner of one
    of the mothers is demanding to see his baby in the special
    care baby unit. He wasn’t present at the birth, and you
    know that the mother hasn’t been in contact with him since
    she became pregnant. What should you do?
    Choose the three most appropriate options for the following
    list:
    A. Take him to see the baby because he is the father
    B. Check the baby’s birth certificate and ask to see some identification
    to confirm who he is
    C. Ask the mother to confirm whether he is the father
    D. Ask the midwife what you should do
    E. Ring the registrar and ask for advice
    F. Document in the notes that this man wanted access to the
    child
    G. Tell the man that he will have to come back another time
    once he has pre-arranged a visit
    H. Tell the man that he cannot see the baby until you have
    spoken to the mother
A

ANSWER: CEH
This is about patient confidentiality and protecting both the
mother and the baby. This is also about recognising parental
rights, and that you cannot refuse a parent access to a
child just because of what the other parent says – providing,
however, that you have confirmed identity.
C. Asking the mother to confirm whether he is the father is one
of the most appropriate options, as you should seek consent from
the mother whether to allow access to her baby – particularly as
the father has been out of the picture for such a long time.
E. Ringing the registrar to ask for advice is appropriate, as you
must consult a senior in tricky situations such as this. There
could be potential problems from either the mother or this man,
and you should alert seniors to this as soon as possible.
H. Telling the man that he cannot see the baby until you have
spoken to the mother is one of the most appropriate options, as
you must explain to him that the mother also has the right to
know who will be visiting her child.
B. Checking the baby’s birth certificate is not one of the preferred
options, as you should always confirm whether the father
is who he says he is. Another way would be to get the mother
to confirm that he is the father.
A. Taking this man to see the baby is not appropriate, as –
given the nature of their relationship – you should not allow
the “alleged” father to see the baby without the mother’s
permission.
D. Asking the midwife for advice is not one of the preferred
options as a nurse has come to you already asking for advice
Answers 81
(rather than the midwife), suggesting that you are the most
appropriate person to handle this situation.
F. Documenting in the notes that this man wanted access to
the baby is also not one of the top selections; however, you
would note this down at a later stage, as well as the conversation
that you had with both parents.
G. Telling the man to come back another time once he has
pre-arranged a visit is not one of the preferred options as
fathers do not have to pre-arrange visits to see their children.
Confirming his identity and getting consent is therefore more
appropriate in this case.

39
Q
  1. A 79-year-old male is awaiting surgical repair of a fractured
    neck of femur. He tells you that he is a Jehovah’s
    Witness and says he will refuse the operation if he needs a
    SJT practice material 63
    blood transfusion. You suspect the risk of needing a blood
    transfusion is high. He asks if there are any other options.
    What should you do?
    Choose the three most appropriate options from the following
    list:
    A. Explain the consequences to him of not having the operation
    B. Ask his permission to contact the local Hospital Liaison
    Committee (HLC)
    C. Advise him to contact the Watchtower Society for
    information
    D. Tell him that you are unsure of the options but that you will
    find out
    E. Take bloods and order routine pre-op investigations
    F. Bleep your registrar to review the patient
    G. Consult your local policy guidance on bloodless medical
    procedures
    H. Find out why he is opposed to a blood transfusion and what
    his preferences are on the use of blood products
A

ANSWER: AHF
This question is about respecting a patient’s wishes and
acknowledging their rights to refuse treatment on the
grounds of religious principles. See Chapter 4 for more
information on Jehovah’s Witnesses.
A. Explaining the consequences of not having the operation is
appropriate. He should fully understand the risks of refusal.
Moreover, there are conservative options but they are also risky.
H. Finding out why he is opposed to a blood transfusion and
his preferences is also appropriate. You need to fully explore
the patient’s personal beliefs before deciding on a management
plan. You can then relay this information to the registrar so that
they are better informed of the patient’s wishes.
F. Bleeping your registrar to review the patient is appropriate,
as you always need senior advice in complex situations involving
patient care. They will know whether there are cell-salvage
facilities at your hospital, or whether he needs to be transferred
to a different hospital. They can also enter into a more complex
discussion with the patient regarding the relative risks and
benefits of the various options available.
D. Telling him you are unsure of the options but you will find
out is not a preferred option as your registrar must review the
patient, in which case they can explain the options to him.
G. Consulting your local guidelines is also not a preferred
option. Although there should be clear guidance on this at your
local trust, this can be accessed after you have all the information
from the patient and passed that on to the relevant senior.
You would not want to delay this patient’s treatment and therefore
should act appropriately to make sure they receive senior
review in a timely manner. Always be aware of where you can
access such policies to read up at a later stage.
B. Asking permission to contact the HLC is not a top-ranked
option. You need to gain consent before you can discuss his
case with an external body. The HLC will be able to give you
information on the options available to him, but this will take
longer than getting an internal senior review where – knowing
the options – it is likely that the HLC need not be contacted.
C. Advising him to contact the Watchtower society is not
appropriate, as the patient does not have enough information
to enquire about such a significant procedure at this stage. It
is your responsibility to be an advocate for this patient and
his care.
E. Taking bloods and ordering pre-op investigations is not
appropriate, as the patient has not consented to the operation,
and therefore these investigations are not considered a necessary
part of their care.

40
Q
  1. A man is sitting in minors in A&E after having a drunken
    brawl with his friend. He has superficial lacerations to his
    shoulder and forearm. He arrived at midnight and, having
    been waiting for three hours, is getting increasingly agitated.
    The nurse comes to tell you that he is angry and threatening
    to self-discharge. You are busy packing the nose of his friend
    whom he was fighting with. What should you do?
    Choose the three most appropriate options from the following
    list:
    A. Allow him to self-discharge, you do not respond to threats
    B. Ask the nurse to let the patient know that he is next
    C. Call security
    D. Apologise to him for the long wait and thank him for his
    patience
    E. Tell the nurse to get the suture kit ready
    F. Ask the nurse to tell him he should wait because you are
    seeing to his friend
    G. Leave his friend and go and attend to his shoulder and
    forearm lacerations
    H. Call the SHO to review and suture him
A

ANSWER: BED
This question is about staying calm whilst under pressure
and maintaining a good relationship with patients through
effective communication. See Chapter 4 for more information
on verbal communication skills.
B. Asking the nurse to let the patient know he is next is the
most appropriate response as it is clear he has been waiting a
while and needs treatment.
E. Telling the nurse to get the suture kit ready is an appropriate
response as this will help you out and save some time. It is
important to recognise when it is appropriate to delegate tasks.
D. Apologising to him for the long wait is an appropriate response
as this shows empathy. Hopefully this will diffuse his agitation and
allow him to be treated. He could very well be agitated because
he is in pain or he doesn’t like hospitals. It is important not to label
patients as “difficult” or “drunk” as this could impair your responsibility
to treat them in their best interests.
A. Allowing him to self-discharge is not appropriate, as you
have a duty of care to your patients. There is no indication that
this patient is going to physically abuse the nurse, or that the
nurse feels threatened in any way. Therefore it is best if they
are persuaded to stay and receive treatment.
C. Calling security is not appropriate at this stage, as he is not
being threatening towards staff or others; he is merely saying
that he will discharge himself against medical advice.
F. Asking the nurse to tell him he should wait because you are
seeing his friend is not appropriate, as it is better to let him
know he is next (B). Although he may be aware that his friend
is also in hospital, this could still be considered a breach of
confidentiality. It is better to keep information as anonymous
as possible.
82 Answers
G. Leaving his friend and going to attend to his shoulder and
forearm lacerations is not appropriate, as you should finish
treating one patient rather than abandoning them to attend to
another patient.
H. Calling the SHO to review and suture him is not appropriate.
You should be able to manage this situation as an FY1. You
should avoid calling the SHO for minor disagreements that
you are more than capable of handling. However, if this was a
laceration to the forehead, you might be concerned that the
increasing agitation was a manifestation of a more serious head
injury, in which case you would consider bleeping the SHO for
a full head injury assessment.

41
Q
  1. You are in the surgical assessment unit, trying to consent
    a 55-year-old man for rigid sigmoidoscopy that you will be
    doing under supervision. You have been trained to consent
    for this procedure and are aware of all the risks and benefits.
    When you try to explain the procedure, he says “do
    whatever you think is best, Doc, I don’t want to know”.
    What should you do?
    Choose the three most appropriate options from the following
    list:
    A. Tell him you need to give him a brief explanation of the
    procedure
    B. Ask him to sign the form without going into detail
    C. Tell him it is important that he understands the procedure
    in order to consent
    D. Phone your registrar to consent the patient
    E. Ask him whether he would prefer a family member with
    him whilst you explain the procedure
    F. Explain the procedure to a family member instead
    G. Cancel the investigation as you cannot go ahead without
    consent
    H. Fill in the form yourself and sign it in his best interests
A

ANSWER: CAE
This is about handling a situation where a patient does not
wish to consent to treatment. See Chapter 3 for more information
on consent in patients with capacity.
C. Telling him it is important that he understands the procedure
in order to consent is appropriate, as you should reiterate
the need for an explanation of the procedure. The patient may
still change his mind about knowing.
A. Telling him you need to give him a brief explanation of the
procedure is appropriate, as you must be honest with your
patient and reason with him as to why he should be involved
in his care. Patients should know the broad nature of the procedure,
as it is possible to proceed without providing all the
information on the risks and benefits.
E. Asking him whether he would prefer to have a family
member with him is appropriate. Even though this man has to
consent to the procedure himself, many patients prefer to have
a relative present. The added benefit of having a relative is that
they are likely to take on board more information than the
patient and can ask questions that the patient otherwise wouldn’t.
This is a way of indirectly engaging the patient in a conversation
about the procedure beyond the brief explanation given to
him and may encourage the patient to ask questions himself.
D. Phoning your registrar to consent the patient is not a preferred
option. You have all the knowledge that the senior has
to consent this patient. Moreover, as you will be doing the
procedure yourself (under supervision), it would be more
appropriate for you to gain the consent. You should try persuading
the patient first, before seeking senior help.
B. Asking him to sign the form without going into detail is not
appropriate, as this is not getting appropriate consent – selecting
this option would question your probity and integrity
F. Explaining the procedure to a family member instead is not
appropriate, as a family member cannot consent for a patient
with capacity and can only do so in a patient who lacks capacity
if they are named as a legal proxy.
G. Cancelling the investigation is not appropriate, as this
response is premature. You should try alternatives before
jumping to cancel an investigation that is designed to help the
patient – a senior would be expected to make this decision.
H. Filling in the form yourself and signing in his best interests
is not appropriate – this is breaking the professional code of
conduct and questions your honesty and professionalism.

42
Q
  1. You are looking after Mrs Chang, who is intermittently
    confused. Her family are all in the waiting room and are
    anxious to know how she is doing. You have the results of
    her CT scan which is normal. What should you do?
    Choose the three most appropriate options from the following
    list:
    A. Go and tell Mrs Chang’s family the good news
    B. Go see Mrs Chang to find out how she is today
    C. Ask a nurse to go and tell the family the good news
    D. Tell Mrs Chang’s family how she is generally doing, but
    say that you cannot give them any specific results until you
    have spoken to Mrs Chang
    E. Wait until Mrs Chang’s family ask specifically about her
    results
    F. Let your colleagues know Mrs Chang’s results
    G. View the scan yourself
    H. Ring the registrar to ask for advice
A

ANSWER: BDH
This is about ensuring patient confidentiality in a patient
who may or may not have the capacity to consent. It is also
about strengthening relationships with relatives and maintaining
good communication.
B. Going to see how Mrs Chang is today is appropriate, as you
should determine whether she has capacity or not. As she is
intermittently confused, you should wait to tell her the news
until she can comprehend it. Telling the family before the
patient is never acceptable – unless you are sure that she is
never going to have capacity.
D. Telling her family about her general well-being without
going into specific details is appropriate. The family will want
to know how she is today and it is important that family
members are kept up to date with her clinical situation. If you
explain that you have to get consent from the patient they
should understand and will appreciate that you have addressed
their anxieties.
H. Ringing the registrar to ask for advice is appropriate, as you
have a duty to relay results to patients in a timely manner. You
will need advice on this matter as it is likely that the family are
worried and will ask you about this. Confidentiality and consent
are complex issues which often extend beyond what you can
see. In this respect, it is always a good idea to get a senior
involved early rather than having to undo a mistake later.
G. Viewing the scan yourself is not a preferred option. Although
it is good practice to look at all scans as well as the radiology
report, Mrs Chang and her family should take priority. Viewing
normal CT scans is just as important as viewing abnormal ones
and this is a learning opportunity, but, ideally, you would need
a senior to talk you through the scan to consolidate information
accurately.
F. Letting your colleagues know Mrs Chang’s result is not a
preferred option. You should let your colleagues know the results
at handover or a more appropriate time. The priority here is to
ascertain how you are going to get the results to Mrs Chang.
Answers 83
E. Waiting until Mrs Chang asks specifically for her results is
not a top selection because you should actively seek to speak
to relatives when results are available – provided you have the
consent from the patient to do so.
A. Telling the family the good news and C Asking a nurse to
tell the family the good news are not appropriate, as the patient
has a right to know their results before the family, regardless
of who delivers the news.

43
Q
  1. You are working on an oncology ward. You are sitting
    at the desk ordering bloods when your colleague Jane tells
    you she thinks that you are not picking up your fair share
    of work. You feel Jane is boisterous and arrogant. No one
    else has mentioned anything to you about being lazy, and
    you feel you are competent. What should you do?
    Choose the three most appropriate options from the following
    list:
    A. Be honest with Jane that you think she is arrogant
    B. Argue with Jane that you do your fair share
    C. Say you cannot discuss this here and suggest a more appropriate
    place
    D. Suggest that you divide the jobs evenly
    E. Ask Jane why she feels you are not doing your share
    F. Raise the issue with your educational supervisor
    G. Ask your other colleagues their opinion of you
    H. Report the bullying to your consultant
A

ANSWER: CED
This is about managing interpersonal relationships and conflict
within a team. You are expected to maintain harmony and
work effectively as a team-player. Refer to Chapter 6 for more
information on teamwork.
C. Saying you cannot discuss this here and suggesting a more
appropriate place is a top selection, as you never want to get
into an argument on the ward in front of patients. This would
be unprofessional.
E. Asking Jane why she feels you are not doing your share is
appropriate, as you should find out from your colleague why
she has a problem with you. That way you can discuss it openly
and hopefully reach a compromise.
D. Suggesting that you divide the jobs evenly is appropriate,
as this means that neither of you can complain about the workload.
Resolving the problem between you is far more appropriate
than involving seniors. You are adults and should be able
to work through this.
H. Reporting the bullying to your consultant and F Raising the
issue with your educational supervisor are not appropriate, as
you should not escalate things at this stage. Try to resolve the
conflict amongst yourselves.
A. Telling Jane you think she is arrogant and B Arguing that
you do your fair share are not appropriate, as neither response
is constructive and will likely lead to more arguments rather
than a solution.
G. Asking your other colleagues their opinion of you is not
appropriate, as involving team members may mean asking colleagues
to take sides. You are expected to maintain harmony
within the team as far as possible.

44
Q
  1. You are working on a busy respiratory ward. One of
    your colleagues, Jack, is consistently lazy to the point where
    he may be compromising patient care. The nurses and ward
    cover at handover have commented on this. What should
    you do?
    Choose the three most appropriate options from the following
    list:
    A. Offer to take on some of Jack’s workload
    B. Ask Jack whether he feels he is struggling
    C. Tell Jack he is not doing his share of the workload
    64 SJT practice material
    D. Inform the foundation programme clinical lead
    E. Recommend the nurse in charge has a quiet word
    F. Ask the nurses to fill in a Clinically Adverse Event (CAE)
    form
    G. Inform your consultant that Jack is struggling
    H. Tell Jack you will speak to your educational supervisor if
    things don’t change
A

ANSWER: BEG
This question is also about teamwork and knowing how to
handle a struggling colleague.
B. Asking Jack whether he feels he is struggling is appropriate,
as you should find out whether your colleague feels there is a
problem. You can then engage in a conversation as to whether
Jack has insight into his behaviour, whether he feels he is struggling,
or whether he is just attempting to get out of his share
of the work.
E. Recommending the nurse in charge has a quiet word is
as appropriate. Given that the nurses have commented on
Jack’s behaviour, it would be appropriate for them to intervene.
Working as part of a multi-disciplinary team, the nurse in
charge should feel they have authority to speak to an FY1 if
they are concerned about their behaviour.
G. Informing your consultant that Jack is struggling is appropriate.
Whilst it is courteous to share this information with your
colleague before your consultant, the question suggests that
patient care is being compromised. Hence, your consultant
needs to be informed of the situation. Moreover, given a few
members of the team have commented, it is right to escalate
this further. If you did nothing, you would be letting down your
colleagues, Jack and the patients under his care.
C. Telling Jack that he is not doing his share of the workload
is not as appropriate. Highlighting your awareness of the situation
to your colleague may produce very little gain in terms
of resolving the problem. Moreover, it may be better that this
information comes from the nurse in charge who has expressed
a concern.
H. Telling Jack you will speak to your educational supervisor
if things don’t change is not a preferred option as your educational
supervisor is only in a position to give you advice and,
unless they happen to be Jack’s educational supervisor too,
cannot help him in the same way that his own educational
supervisor should.
A. Offering to take on some of Jack’s workload is not appropriate.
This would increase pressure on you, in an already busy
situation. Moreover, this is not addressing the real issue. You
could always help them out if you have finished your jobs, but,
ideally, they need to seek help from their educational supervisor
if they cannot cope.
D. Informing the foundation programme clinical lead is not
appropriate, as there are more suitable reporting channels.
F. Asking the nurses to fill in a CAE form is not appropriate,
as you should speak Jack before filling in a CAE form regarding
his conduct. This would also be escalated outside the team
and Jack would be referred to see the foundation programme
clinical lead. There is no indication in the question of one
specific incident compromising care that would require filling
in this form.

45
Q
  1. You send a medical student to take some bloods – for
    which they are trained. You check the results at 3 p.m.
    and realise that the samples are all coagulated. What should
    you do?
    Choose the three most appropriate options from the following
    list:
    A. Retake the bloods
    B. Report the student to your consultant
    C. Point out the mistake to the student when you see them the
    following day
    D. Ignore their mistake
    E. Supervise the student when taking blood next time
    F. Tell the student to bring the samples to you next time before
    they send them
    G. Suggest the student reflects on this incident in their portfolio
    H. Tell the student to get some more clinical skills training
    before they go on the wards
A

ANSWER: AEG
This question is about teaching medical students to take
responsibility for their own actions and own up to mistakes.
A. Retaking the bloods is appropriate, as the patient comes
first, and the mistake must be corrected.
E. Supervising the student when taking bloods next time is
appropriate, as the question indicates an issue with the samples
taken. It would be helpful for you to go with the student to
ensure they are doing the job correctly and to give them practical
advice. You can also take this opportunity to discuss with
them the difficulties associated with venepuncture.
G. Suggesting the student reflects on this incident in their portfolio
is appropriate. Reflecting means the student can learn about
the consequences of not checking what they have done is right,
and the implications for the patient. This will be an important
learning point for the student which they should not ignore.
C. Pointing out the mistake to the student is not a preferred
option. Although you are responsible for teaching and learning,
it would be better to supervise them where a problem has
occurred rather than highlighting the error and moving on.
F. Telling the student to bring the sample to you next time is also
not a top selection, as it would be better to supervise the student
rather than just checking that the bloods are OK before sending
them to the laboratory. It is good practice to check them regardless
– especially if it is for something like a blood transfusion.
B. Reporting the student to your consultant is not appropriate, as
this puts the student in an awkward position and you don’t want
to scare them away from attempting venepuncture ever again.
D. Ignoring their mistake is not appropriate, as the student will
not learn if it is ignored.
H. Telling the student to get some more clinical skills training
before they go onto the wards is not appropriate, as the best
practice involves real patients. They have already been trained
in venepuncture and telling them to go back to square one will
dent their confidence further. You should be supportive towards
students to most effectively enable their learning.

46
Q
  1. You are working in a GP surgery. A patient comes in to
    see you with symptoms of angina at rest. It says on the
    system that the GP told them last time not to drive. You ask
    them whether they have been driving, and they tell you that
    they have: they drove to the surgery today. What should
    you do?
    Choose the three most appropriate options from the following
    list:
    A. Report them to the DVLA
    B. Advise them to inform the DVLA
    C. Advise them to stop driving until their symptoms are under
    control
    D. Find out whether someone can drive them home
    E. Ask if they remember the GP advising them not to drive
    F. Remind them last time the GP asked them not to drive
    G. Speak to the GP for advice
    H. Ask them to hand over their keys until someone can drive
    them home
A

ANSWER: ECG
This question tests your knowledge of effective communication
concerning patient guidance. See Chapter 2 for more
information on the DVLA.
E. Asking if they remember the GP advising them not to drive
is appropriate, as you should reiterate that you know they have
been given this information already. This is more polite than
option F.
C. Advising them to stop driving until their symptoms are under
control is appropriate, as you should relay the advice they have
already received about not driving with symptoms of angina.
G. Consulting the GP for advice is appropriate. Although you
should be aware of the guidance and can give patient the appropriate
advice (C), this is a tricky situation and you should seek
senior help on how to proceed.
D. Finding out whether someone can drive them home is not
a preferred option as you should find out from the GP how best
to proceed with getting the patient home. This would, however,
establish whether someone could come and collect them to
drive them home.
A. Reporting them to the DVLA is not appropriate, as you
should not inform the DVLA without the patient’s consent.
Also, it is the patient’s responsibility to do this, if necessary.
B. Advising them to inform the DVLA is not appropriate, as it
may be that the patient misinterpreted the advice given by the
GP. Also, the DVLA guidance suggests that driving must cease
until symptoms are under control but they need not be notified
for angina.
F. Reminding them that last time the GP asked them not to
drive is not a preferred option as it is more aggressive than E
and the patient may have forgotten they were asked not to drive.
H. Asking them to hand over their keys is not appropriate, as,
once their symptoms are adequately controlled, they should be
allowed to drive. It would be advisable for someone to pick them
up, but if you are so worried you have to confiscate their keys
– they probably should be going to hospital rather than home.

47
Q
  1. Your consultant on a ward round orders a spine MRI
    and asks you to put in the request. You overhear the registrar
    saying to another colleague that it is not indicated.
    What should you do?
    Choose the three most appropriate options from the following
    list:
    A. Order the MRI as the consultant has requested
    B. Ask the consultant the reasons for ordering the MRI
    C. Ignore the consultant and do as your registrar says
    D. Tell the consultant that the registrar does not think the MRI
    is indicated
    E. Ask the registrar why they think the MRI is not indicated
    F. Ring the radiologist to discuss the MRI request
    G. Ask the registrar to speak to consultant about their difference
    in opinion
    H. Document in the notes that the MRI was ordered but that
    the registrar disagreed
A

ANSWER: BEF
This question is about managing conflicts within a team and
maintaining harmony whilst making sure the more appropriate
action is taken.
B. Asking the consultant the reasons for ordering the MRI is
appropriate, as this could simply be an opportunity for learning
about indications for a particular scan. Moreover, you should
be putting this on the request form.
E. Asking the registrar why they think the MRI is not indicated
is appropriate, as this would be an opportunity for conflict resolution.
If you find out the reasons that the registrar disagrees
with the indication, you can explain clearly why the consultant
ordered the MRI.
F. Ringing the radiologist to discuss the MRI request is appropriate.
Radiologists are busy, but they would appreciate you
politely ringing to discuss an MRI request. Moreover, you can
include any indications you discuss on the form which will help
Answers 85
the radiographer select the best images. The radiologist could
also raise concerns if they felt that the scan wasn’t indicated,
in which case you could relay that information back to your
consultant.
A. Ordering the MRI as the consultant has requested is not
appropriate. Although you should never undermine your consultant,
you cannot ignore the comments of your registrar.
Patient care should come before disagreements, and patients
should only have scans when indicated.
C. Ignoring the consultant and following your registrar is
rarely appropriate, if ever. You should not ignore your consultant
or go against their decision without their knowledge. In the
event there was a change to their management plan and they
could not be contacted, you would have to document everything
in the notes including the arguments on both sides and
why the final decision was made. Under no circumstances
should you deliberately undermine your consultant.
D. Telling the consultant that the registrar does not think the
MRI is indicated is not appropriate, as you have a duty not to
create conflict within your team.
H. Documenting in the notes that the MRI was ordered but that
the registrar disagreed is not a preferred option. Ideally you
should not document disagreements within the team in the
notes – unless they are directly relevant to patient safety.

48
Q
  1. You are working as an FY1 as part of the medical oncall
    team. A patient with type I diabetes came in with
    diabetic ketoacidosis (DKA), but is improving having been
    on a sliding-scale. The consultant prescribes short-acting
    insulin. The patient disagrees with this and asks that he be
    put back onto his regular insulin regimen of long-acting
    insulin. What should you do?
    Choose the three most appropriate options from the following
    list:
    A. Ask the specialist diabetic nurse for advice
    B. Call the consultant to relay the patient’s disapproval of their
    regimen
    C. Prescribe the patient long-acting insulin instead
    D. Tell the patient that they are on the right insulin
    E. Tell the patient they will have to discuss it with the consultant
    the next time they see him
    F. Write both up on the drug chart to be delivered
    G. Ask the patient why they don’t want to have short-acting
    insulin
    H. Ignore the patient’s request as they are clearly still unwell
    and confused
A

ANSWER: AGB
This question is about advocating for your patient and utilising
expert knowledge within your team.
A. Asking the specialist diabetic nurse for advice is appropriate,
as they have a wealth of knowledge regarding the prescribing
of insulin and, if needs be, can act as another advocate for
the patient using their expertise.
G. Asking the patient why they don’t want to have short-acting
insulin is appropriate, as you want to find out the patient’s
wishes before you can discuss them with the senior in
charge.
B. Calling the consultant to relay the patient’s disapproval of
their regimen is also appropriate, as you must – at all times – be
an advocate for your patient and meet their wishes as far as
possible. If a patient is unhappy with their care, you have a duty
to let your consultant know about it – tactfully!
C. Prescribing long-acting insulin D Telling the patient they
are on the right insulin and H Ignoring the patient’s request
are not appropriate, as these options all ignore the patient’s
wishes and their rights to be involved in treatment.
E. Telling the patient they will have to discuss the option with
the consultant is not appropriate, as this postpones the issue
and it is your job to broach the subject with the consultant, not
theirs.
F. Writing both up on the drug chart to be delivered is not
appropriate, as this leaves potential for a prescribing error if
it is not clear which insulin should be delivered to the patient
and when.

49
Q
  1. A 69-year-old man is brought into A&E with symptoms
    later confirmed as an ischaemic stroke. Your specialist registrar
    reviews him in A&E and writes up his prescription
    on a drug chart. The patient reaches you on the stroke ward
    one hour later with his notes and a different drug chart.
    Clopidogrel is written up STAT but not given. The prescription
    written is not the one that your registrar wrote. What
    should you do?
    Choose the three most appropriate options from the following
    list:
    A. Ring pharmacy to ask for advice
    B. Bleep your registrar
    C. Take the chart down to A&E and look for the other one
    D. Ask the patient whether they have been given the
    Clopidogrel
    E. Give the Clopidogrel as it is important they receive urgent
    anticoagulation
    F. Give half the dose of Clopidogrel
    G. Delay giving the Clopidogrel until the other chart is
    found
    H. Tell the nurses not to give Clopidogrel to the patient
A

ANSWER: BCG
This is about recognising the potential for a prescribing
error which could potentially harm the patient. Remember
not to PANIC when prescribing: Prescription, Allergy,
Notes, Interactions, Clear. This error involves the prescription,
i.e. ensuring that only one dose of the medication is
given (where 85% of errors occur), and the notes i.e. two
drug charts. See Chapter 3 for more details.
B. Bleeping your registrar is appropriate, as you need to find
out what happened to the other drug chart and whether the
patient has received their Clopidogrel.
C. Taking the chart down to A&E whilst looking for the other
one is appropriate, as you can ask the nurses where this patient
was and whether they know anything about the prescription as
well as having a look for the drug chart. This should quickly
resolve the situation and, as a matter of urgency, should be your
responsibility.
G. Delaying giving the Clopidogrel until the other chart is
found is appropriate. Although nearly all patients receive anticoagulation
in hospital for deep vein thrombosis (DVT) prophylaxis,
you would not want to put them at risk of an intracranial
bleed by titrating them outside of the therapeutic range. Given
this patient has had an ischaemic stroke, this situation does need
to be resolved promptly, but delaying is the safer option.
A. Ringing the pharmacy to ask for advice is not appropriate.
It is unlikely that pharmacy will be able to give you any indication
of whether the patient has received the Clopidogrel or not.
They can provide advice on the correct protocol – but these
options are available. This question tests your knowledge of
safe practice.
D. Asking the patient whether they have been given the Clopidogrel
is not appropriate, as the patient may not have an idea
of what they have been given and you could be given false
information. It is best to rely on the facts – which means finding
both charts.
86 Answers
E. Giving the Clopidogrel and F Giving half the dose of Clopidogrel
are not appropriate, as they both put the patient at risk
of bleeding. Medication should not be changed or given without
having all the accurate and relevant information.
H. Telling the nurses not to give Clopidogrel to the patient is
not a preferred option. Although communication is key to preventing
errors, the nurses would not give the prescription
without seeing the chart (nor if you crossed it out). If you take
the chart down with you to A&E, this will prevent the patient
receiving the medication.

50
Q

You are working on a busy cardiology ward. Your colleague
is on call and asks you to hold their bleep for them whilst
they go and get some lunch. What should you do?
Choose the three most appropriate options from the following
list?
A. Agree to hold their bleep
B. Offer to go with them for their lunch
C. Politely decline because you have your own patients
D. Suggest they get in touch with someone on the on-call team
to hold their bleep
E. See whether they have any outstanding jobs you can help with
F. Offer to get their lunch for them
G. Tell them to turn it off whilst they go and get some lunch
H. Suggest they go quickly to get their lunch

A

ANSWER: CEF
This question is about trying to help out your colleague but
recognising that it is not appropriate to pass over your bleep.
C. Politely declining because you have your own patients is
appropriate, as you should not take someone else’s bleep when
you are not on-call. You have a responsibility for your own
patients and they have to be your priority. Taking on another
workload on an already busy ward is not acceptable.
E. Seeing if they have any outstanding jobs you can help with
is appropriate, as this is offering your colleague support so they
can find some time to go and get their lunch. That way you stay
on the ward without taking on any new patients.
F. Offering to get their lunch for them is appropriate, as this
will be helping out your colleague by saving them time so they
can take a shorter break to sit down and eat it.
A. Agreeing to hold their bleep is not appropriate. You should
not hold someone else’s bleep – unless under exceptional circumstances
such as breaking bad news.
B. Offering to go with them for their lunch is not appropriate.
If you want to help your colleague out, you should stay
on the ward and go for your own lunch when you get an
opportunity.
D. Suggesting they get in touch with someone from the on-call
team to hold their bleep is not appropriate. You should not give
your bleep to anyone else. More importantly, if your colleague
is busy, the other members of the on-call team are also likely
to be busy.
G. Telling them to turn it off whilst they go and get some lunch
is not appropriate. There might be a sick patient whom your
colleague needs to attend to, and therefore not being accessible
could compromise patient care.
H. Suggesting they go quickly to get their lunch is not appropriate,
as this does not help out your colleague.